Vous êtes sur la page 1sur 108

1

Nursing Entrance
Exam Practice
Tests
2

Table of Contents

Table of Contents................................................................................................2
Section 1 Mathematics (Practice Test 1)...........................................................3
Answer Key for Mathematics Section 1 (Practice Test 1)............................................................... 10
Section 2 Verbal (Practice Test 1) ...................................................................12
Section 2 Reading Comprehension (Practice Test 1) ....................................17
Answer Key for Word Knowledge Section 2 (Practice Test 1) ....................................................... 26
Answer Key for Reading Comprehension Section 2 (Practice Test 1) ........................................... 27
Section 3 Science (Practice Test 1).................................................................28
Answer Key for Science Section 3 (Practice Test 1)......................................37
Section 1 Mathematics ( Practice Test 2)........................................................39
Answer Key for Mathematics Section 1 ( Practice test 2) ............................................................... 45
Section 2 Verbal (Practice Test 2) ...................................................................47
Section 2 Reading Comprehension (Practice Test 2 ) ...................................52
Answer Key for Word Knowledge Section 2 (Practice Test 2) ....................................................... 62
Answer Key for Reading Comprehension Section 2 (Practice Test 2) ........................................... 63
Section 3 Science (Practice Test 2).................................................................64
Answer Key for Science Section 3 ( Practice Test 2) ....................................................................... 73
Section 1 Mathematics (Practice Test 3).........................................................75
Answer Key for Mathematics Section 3 (Practice Test 3)............................................................... 81
Section 2 Verbal (Practice Test 3) ...................................................................83
Section 2 Reading Comprehension (Practice Test 3) ....................................88
Answer Key for Word Knowledge Section 2 (Practice Test 3) ....................................................... 96
Answer Key for Reading Comprehension Section 2 (Practice Test 3) ........................................... 96
Section 3 Science (Practice Test 3).................................................................98
Answer Key for Science Section 3 ( Practice Test 3) ..................................................................... 107
3

Section 1 Mathematics (Practice Test 1)

Work each problem carefully. You are allowed to use scrap paper to do
your calculations. The correct answers are found after the section. You are
allowed 45 minutes to complete this test.

1. 25% of 400 =
A. 100
B. 200
C. 800
D. 10,000

2. 22% of $900 =
A. 90
B. 198
C. 250
D. 325

3. Which of these numbers is a factor of 21?


A. 2
B. 5
C. 7
D. 42

4. (9÷3) x (8÷4) =
A. 1
B. 6
C. 72
D. 576

5. Once inch equals 2.54 cm. How many centimeters tall is a 76- inch man?
A. 20 cm
B. 29.92 cm
C. 193.04 cm
D. 300.04 cm

6. What is the reciprocal of 6?


A. ½
B. 1/3
C. 1/6
D. 1/12

7. A room measures 11 ft x 12 ft x 9 ft. What is the volume?


A. 1188 ft3
B. 32 ft3
C. 120 ft3
D. 1300 ft3
4

8. A roast was cooked at 325 °F in the oven for 4 hours. The internal
temperature rose from 32 °F to 145 °F. What was the average rise in
temperature per hour?
A. 20.2 °F/hr
B. 28.25°F/hr
C. 32.03°F/hr
D. 37°F/hr

9. You need to purchase a textbook for nursing school. The book cost $80.00,
and the sales tax where you are purchasing the book is 8.25%. You have
$100. How much change will you receive back?
A. $5.20
B. $7.35
C. $13.40
D. $19.95

10. You purchase a car making a down payment of $3,000 and 6 monthly
payments of $225. How much have you paid so far for the car?
A. $3225
B. $4350
C. $5375
D. $6550

11. Your supervisor instructs you to purchase 240 pens and 6 staplers for the
nurse’s station. Pens are purchase in sets of 6 for $2.35 per pack. Staplers
are sold in sets of 2 for 12.95. How much will purchasing these products
cost?
A. $132.85
B. $145.75
C. $162.90
D. $225.05

12. Which of the following percentages is equal to 0.45?


A. 0.045%
B. 0.45%
C. 4.5%
D. 45%

13. A vitamin’s expiration date has passed. It was suppose to contain 500 mg of
Calcium, but it has lost 325 mg of Calcium. How many mg of Calcium is left?
A. 135 mg
B. 175 mg
C. 185 mg
D. 200 mg
5

14. You have orders to give a patient 20 mg of a certain medication. The


medication is stored 4 mg per 5-mL dose. How many milliliters will need to be
given?
A. 15 mL
B. 20 mL
C. 25 mL
D. 30 mL

15. In the number 743.25 which digit represents the tenths space?
A. 2
B. 3
C. 4
D. 5

16. Which of these percentages equals 1.25?


A. 0.125%
B. 12.5%
C. 125%
D. 1250%

17. If the average person drinks 8, (8oz) glasses of water per day, a person who
drinks 12.8 oz of water after a morning exercise session has consumed what
fraction of the daily average?
A. 1/3
B. 1/5
C. 1/7
D. 1/9

18. If y = 3, then y3(y3-y)=


A. 300
B. 459
C. 648
D. 999

19. 33% of 300 =


A. 3
B. 9
C. 33
D. 99

20. You need 4/5 cups of water for a recipe. You accidentally put 1/3 cups into
the mixing bowl with the dry ingredients. How much more water in cups do
you need to add?
A. 1/3 cups
B. 2/3 cups
C. 1/15 cups
6

D. 7/15 cups

21. ¾ - ½ =
A. ¼
B. 1/3
C. ½
D. 2/3

22. You cannot find your 1 cup measuring cup. You can only locate your ¼
measuring cup. Your recipe calls for 2 ½ cups of flour. How many times will
you need to fill your ¼ measuring cup with flour for the recipe?
A. 4
B. 6
C. 8
D. 10

23. You are financing a computer for $5000. You are required to put down a 15%
down payment. How much money do you need for your down payment?
A. $500
B. $650
C. $750
D. $900

24. You are travelling in Europe, and you see a sign stating that London is 3
kilometers away. If 1 kilometer is equal to 0.625 miles, how many miles away
is London from where you are?
A. 0.208 miles
B. 1.875 miles
C. 2.75 miles
D. 3 miles

25. You need exactly a 1680 ft3 aquarium for your fish. At the pet store you see
four choices of aquariums, but the volume is not listed. The length, width, and
height are listed on the box. Which of the following aquariums would fit your
needs?
A. 12 ft x 12 ft x 12 ft
B. 13 ft x 15 ft x 16 ft
C. 14 ft x 20 ft x 6 ft
D. 15 ft x 16 ft x 12 ft

26. You invested 9,000 and received yearly interest of $450. What is your interest
rate on your investment?
A. 5%
B. 6%
C. 7%
D. 8%
7

27. In your class there are 48 students, 32 students are female. Approximately
what percentage is male?
A. 25%
B. 33%
C. 45%
D. 66%

28. If w = 82 +2, and z = 41 (2), then


A. w<z
B. w>z
C. w-z = 1
D. w=z

29. After talking with his girlfriend on the telephone long distance, a student
calculates the amount of money he spent on the call. The first 20 minutes
were 99 cents, and each additional minute was 10 cents. He calculated that
his phone call cost $ 5.49. How long was his call?
A. 40 minutes
B. 45 minutes
C. 65 minutes
D. 75 minutes

30. You are teaching a community education class on fire safety to children.
There are 6 first graders, 7 second graders, and 5 third graders. What
percentage of the class is second graders?
A. 28%
B. 33%
C. 39%
D. 48%

31. One slice of bread is 80 calorie. Approximately how many calories are in 2 ½
slices of bread?
A. 140 calories
B. 200 calories
C. 220 calories
D. 240 calories

32. 7x = 3a + 2a. If a = 7, then x =


A. 5
B. 7
C. 9
D.12

33. If the rule for computing an infant’s dose of medication is:


Child’s age in months
8

infant’s dose = ------------------------------- X adult dose


150

If the adult dose of medication is 15 mg, how much should be given to a 2


year-old child?
A. 1.2
B. 2.4
C. 3.6
D. 4.8

34. What is the area of a triangle if the base is 6 cm and the height is 8 cm?
A. 18 cm3
B. 20 cm3
C. 22 cm3
D. 24 cm3

35. 7 ½ - 5 3/8 =
A. 1 ½
B. 1 2/3
C. 2 1/8
D. 3 ¼

36. The school’s softball team won 15 games, but lost 10. What was ratio of wins
to losses?
A. 2:1
B. 3:1
C. 3:2
D. 4:1

37. 35 is 20% of what number?


A. 175
B. 186
C. 190
D. 220

38. 6 x 0 x 5
A. 30
B. 11
C. 25
D. 0

39. 7.95 ÷ 1.5


A. 2.4
B. 5.3
C. 6.2
D. 7.3
9

40. If x = 4, then 2 x + 7x =
A. 24
B. 27
C. 35
D. 36
10

Answer Key for Mathematics Section 1 (Practice Test 1)

1) A ( 400 x .25) = 100


2) B ($900 x .22) = 198
3) C (Factors are numbers that can divide evenly into a certain number. 7 can
divide into 21 three times.)
4) B (3) x (2) = 6
5) C ½.54 = 76/x , 1x = 2.54 (76), x= 193.04
6) C The reciprocal of a fraction is the inverse of the fraction. The fraction is
turned up side down. 6 = 6/1, the reciprocal is 1/6.
7) A 11 ft x 12ft x 19 ft = 1188 ft3
8) B 145°F-32°F = 113°F, 113°F ÷4hrs = 28.25°F/ hr
9) C $80 x 0.825= $6.60, $ 80+ $6.60= $86.60, $100-$86.60= $13.40
10) B $3000 + ($225(6 payments)= $4350
11) A 240÷6 = 40, 6÷2=3, (40 pcks of pens x $2.35 ea.) + (3 pcks of staplers x
$12.95 ea.) =$132.85
12) D .45 x 100 = 45%
13) B 500mg Ca – 325 mg Ca = 175 mg Ca
14) C set up ration of 4/5 = 20/x, 4x= 20(5), x = 100/4, x= 25 mL dose
15) A moving right of the decimal point one space is the tenths position
16) C 1.25 x 100= 125%
17) B 12.8 ÷ 64 = .20. .20= 20/100, reduce 20/100 to 1/5
18) C 27( 27-3)= 27(24) = 648
19) D 300 x .33= 99
20) D 4/5 –1/3, find common denominator, 12/15-5/15= 7/15 cups
21) A ¾ - ½ = ¾ - 2/4 = 1/4
22) D 2 ½ converts to 5/2, find common denominator 10/4 ÷ ¼ = 10 cups
23) C $5000 x .15 = $750
24) B .625 x 3 = 1.875 miles, you can also set up as a ratio
25) C 14 ft x 20 ft x 6 ft = 1680 ft3
26) A 450 ÷ 9000 = .05, .05 x 100= 5%
27) B 48 students – 32 girls = 16 boys, 16 ÷ 48= .33, .33 x 100=33%
28) B w=84, y = 82, 84>82
29) C .99 + .10 (x) = $5.49, .10 x= 5.49-.99, x= 4.50/.10, x= 45, 45+20=65 mins
(Don’t forget the original 20 minutes from the flat rate of 99 cents.)
30) C 7÷18= .38, .38 x 100 = 38%
31) B 80 calories x 2.5 slices of bread = 200 calories
32) A 7x= 21+14, 35 ÷ 7 = x, x=5
33) B infant dose = (24 months ÷ 150) 15 mg= 2.4mg
34) D ½ (b) (h) = ½ (6) (8)= 24 cm3
35) C (15/2) – (43/8)= (60/8)- (43/8)= 17/8 = 2 1/8
36) C 3:2 both 15 and 10 are divisible by 5
37) A 20% is 1/5 of a number 1/5 = 35/x. 1x= 35( 5), x = 175
38) D any number multiplied by 0 is equal to 0
11

39) B 7.95 ÷1.5= 5.3, remember to move decimal point one place over before
beginning equation
40) D 2(4) + 7(4) = 8 + 28= 36
12

Section 2 Verbal (Practice Test 1)

Word Knowledge: Read each sentence carefully. Then, on the basis of what
is stated in the sentence, select the correct completion of the incomplete
statement. The correct answers will be found at the end of this section.

1. John prefers contemporary art to the classics.


Contemporary means
A. Fashionable
B. Modern
C. Classless
D. Simple

2. Allen told Steve that he would give him the unabridged version of his morning
when he had time. Unabridged means.
A. Abbreviated
B. Funny
C. Linked
D. Complete

3. Lisa was known for having promiscuous relationships.


Promiscuous means
A. Open
B. Friendly
C. Casual
D. Prolific

4. Danny complained of a blind spot on his eye. It was a


A. Scrag
B. Scotoma
C. Riyal
D. Roan

5. The new tax was passed for revitalizing the waterfront district.
Revitalizing means
A. Restoring
B. Policing
C. Revealing
D. Recycling

6. The increased revenue to the class fund allowed for an end of the year party.
Revenue means
A. Donations
B. Support
13

C. Expenses
D. Income

7. The teenager pilfered some candy from the grocery store.


Pilfered means
A. Ate
B. Stole
C. Borrowed
D. Bought

8. Being from a small town, some of Dean’s views were parochial.


Parochial means
A. Light-hearted
B. Threatening
C. Narrow
D. Distant

9. All of the students dreaded the quizzes the professor gave since he tested on
obscure material.
Obscure means
A. Inconspicuous
B. Difficult
C. Well-known
D. Tedious

10. Sally loved to have the latest styles in clothing and would sometimes wear an
outfit only once. Some people might call her
A. Thrifty
B. Obnoxious
C. Oblique
D. Frivolous

11. The judge sued the newspaper for libel.


Libel means
A. False publishing
B. Excessive advertising
C. Occult practices
D. Useless information

12. The farmer complained that his tractor went kaput.


Kaput means
A. Flat
B. Out
C. Dead
D. Incapacitated

13. The hues of the rainbow were vivid against the bright blue sky.
14

Hue means
A. Alabaster
B. Curve
C. Color
D. Line

14. The president has a hedge of protection around him when he makes public
appearances.
Hedge means
A. Leafy bush
B. Barrier
C. Cone
D. Derrick

15. Being late to a party was Aaron’s hallmark.


Hallmark means
A. Tardy
B. Bad habit
C. Calamity
D. Distinguishing trait

16. A small selection of terms was found at the back of the textbook. It was a
A. Glossary
B. Preface
C. Diction
D. Kefir

17. The horror movie frightened the children. It was


A. Melancholy
B. Dramatic
C. Ghastly
D. Tragedy

18. After practice, the girl’s softball team stated, “We’re famished!”
Famished means
A. Fatigued
B. Hungry
C. Excited
D. Ready

19. The newborn baby was enamored with the rattle.


Enamored means
A. Fascinated
B. Happy
C. Unsure what to do
D. Aggravated
15

20. When having a problem, it is best to dissect the situation then act.
Dissect means
A. Cut apart
B. Talk about
C. Ignore
D. Analyze

21. The books subject matter was loathsome to the masses, and it did not sell.
Loathsome means
A. Distasteful
B. Boring
C. Disgraceful
D. Colloquial

22. The kitten was soaked to the skin from the storm. It was
A. Bedraggled
B. Adorable
C. Frightened
D. Craven

23. The bouncer’s countenance discouraged brawls.


Countenance means
A. Message
B. Presence
C. Expression
D. Strength

24. The child apprized her father’s authority and behaved herself in church.
Apprized means
A. Appreciated
B. Compromised
C. Defied
D. Noted

25. David was known for belching and telling inappropriate jokes in public. His
behavior was considered
A. Brusque
B. Terse
C. Critical
D. Uncouth

26. Graduation from high school is considered by many a momentous occasion.


This would be an
A. Azide
B. Epoch
C. Industry
16

D. Ode

27. Nurses play a vital role in the healthcare profession.


Vital means
A. Essential
B. Reoccurring
C. Wonderful
D. Yauld

28. After having his tonsils removed, the child was listless for a few days.
Listless means
A. Hurting
B. Playful
C. Awake
D. Lethargic

29. The park was serene at twilight


Serene means
A. Dangerous
B. Peaceful
C. Dark
D. Noisy

30. The patient’s mind was lucid during the evaluation.


Lucid means
A. Clear
B. Confused
C. Agitated
D. Blank
17

Section 2 Reading Comprehension (Practice Test 1)

There are five reading passages in this section. Read each passage carefully.
Then, on the basis of what you have read in the passage, select the best answer
for each question

Reading 1

Most brides and grooms over spend when they plan their wedding. They choose
the most expensive items and do not set a realistic budget. Most couples forget
that it is only one day in their life. Here are a few tips on how you can avoid the
wedding money mistake that most couples make.

Wedding budgets can range from one hundred dollars to a life long debt that may
never be retired. Your job is to set a firm budget and stick with it. If you spend
more in one area, you must cut in another one.

Once you decide your budget, here are a list of items and percents of the budget
that they will take. Wedding and engagement rings will require 15 percent of your
budget. The wedding apparel will cost approximately 6 percent of your
expenditures. The reception, food, and wedding cake will take 42 percent of your
budget. Invitations and decorations including flowers will eat up 17 percent. The
officiant, limousine rental, photographer and videographer will cost another 13
percent of your budget. The wedding party gifts, rehearsal dinner, and pre-
wedding parties will take up the rest.

Do not make the same mistake many naive brides do by placing wedding debt
onto their credit card. If your credit card debt is the same each month or
increasing with every month, it may take 40 years for you to pay off your wedding
especially if you have an 18% APR. This is also exclusively your debt; you are
stuck with the bill if your ex-fiancé decides to skip town the night before the
wedding.

Assess your current cash reserves. Do not count on gifts to cover the cost of
your wedding. If you need to cash in CD’s, bonds, or stocks to pay for your
wedding, cash them in well in advance. Most service-oriented companies like to
have their cash in hand before the big day.

Set up a separate checking account for your wedding so that you will not
overdraw your personal account and spend more than you intended. With that in
mind, put ten percent more than you intended to use on your budget into that
checking account. In the event that you do go over budget, you will not be hit
with insufficient funds fees.

Stay away from short-term loans offered by many financial institutions. The fees
are often hidden and can quickly accumulate to more than you borrowed. These
18

loans are very difficult to repay in the short amount of time given. If one payment
is missed, you will have a substantial late fee tacked on top of your loan balance.

To save money after the wedding, be sure to have someone responsible for
returning all rental items before their due date. Some companies have substantial
late fees that can be the equivalent to the cost of the rental. They feel that if you
do not return it then they cannot rent it.

Remember that this is one day of your life. Ask yourself if the debt you
accumulate is worth the stress on your new marriage.

31. This passage is mainly about


A. Coping with marriage problems
B. Saving money for your wedding
C. Avoiding marriage pitfalls
D. Budgeting for your wedding

32. We may infer from the article that a wedding is not


A. Worth going into lots of debt.
B. The joyful occasion it is usually thought to be.
C. An expensive party.
D. That costly.

33. The article states that the wedding gifts, rehearsal dinner, and pre-
wedding parties will take up the rest. What is this sentence stating?
A. These items will use up the rest of the budget.
B. These are necessary parts of a complete wedding.
C. These events and items will be the most costly part of your wedding.
D. These do not need to be included in the budget.

34. What percent do the above stated items take up of the budget?
A. 4%
B. 5%
C. 7%
D. 8%

35. The article states that most brides and grooms “do not set a realistic budget.”
Realistic means
A. Significant
B. Practical
C. Firm
D. Ideal

36. We may infer from the article that many weddings


A. Are budgeted.
19

B. Go over budget.
C. Are exactly what the bride want.
D. Do not have to cost more than one hundred dollars.

Reading 2

Dogs are amazing creatures designed with a unique blend of senses. These
senses are similar to humans but are found in different concentrations in dogs.

A dog’s sense of sight can detect movement at greater distances than humans
can. This occurs because their eyes are wider apart than human eyes, but they
have difficulty seeing closer objects. They are able to see in dimmer light that
helps them with night hunting. Dogs see few colors and can only distinguish
between a few of them.

Dogs are known for their sense of smell. A dog can follow a trail that is weeks
old. They are able to smell scents that are one-millionth the concentration that
humans require to detect an odor. This trait makes them beneficial as police
dogs. They can be trained to detect drugs, locate dead bodies, or find lost
children.

A dog’s sense of hearing is amazing; dogs can hear approximately four times
better than humans can. Dogs similar to humans have a wide range of
vocalizations that include whimpers, moans, whines, and barks.

Dogs have fewer taste buds than human have. Dogs are willing to eat or chew on
about anything. They are omnivores since they eat both meat and plant material.
Moreover, dogs’ sense of touch is similar to a human’s touch. They enjoy being
petted, groomed, and played with by their owners.

Dogs have the same senses as human, but they are found in different
concentrations.

37. Which of these titles would describe best this passage?


A. “Making Sense of a Dog’s Senses”
B. “A Dog’s Life”
C. “Dogs Save Lives with Their Senses”
D. “Canine’s Plight”

38. We can infer from the article that a dog could hear
A. Better than a cat.
B. The same as a human.
C. Worse than a human.
20

D. Better than a human.

39. The dog is able to smell what concentration of an odor compared to a


human?
A. one-tenth
B. one-thousandth
C. one-millionth
D. one-billionth

40. What sense that a dog has would be most beneficial to the police
department.
A. sight
B. taste
C. touch
D. smell

41. The article states that “Dogs see few colors and can only distinguish
between a few of them.”
Distinguish means
A. Make important
B. Design
C. Concern
D. Differentiate

42. As described in the article, which is the best statement to describe, what
dogs eat?
A. Dogs eat meat and vegetables.
B. Dogs are solely carnivores.
C. Dogs exclusively eat plant material.
D. Dogs eat only meat.

Reading 3

Frank Tower, have you ever heard of him? He is the questionable figure who
supposedly survived three doomed ships in the 1900’s. Some consider him one
of the luckiest men alive.

He was touted to be a middle-aged fireman in the engine room. Some


considered him an ordinary, hardworking person, but he had the ability to avoid
dying in some of the most horrendous ocean liner accidents ever recorded.

He was said to have once been a crewmember on the Titanic at the time that the
ship hit the iceberg. Two years later, he was working on the Empress of Ireland
when she collided with the Storstad. Over one thousand people died in that
21

disaster. He was then employed in May of 1915 on the Lusitania when it was hit
by a U-20 torpedo. He apparently lived through that without a scratch as well.
If you are beginning to doubt this man’s existence, you are probably not to far
from the truth. No records have been found ever listing a man by Frank Towers
working on any of the three ships.

The legend of Frank Towers seems to be another case of an urban folk tale,
humanity’s desire to see triumph over a tragic situation. Fact or fiction, Frank
Towers is one of the multiple characters that help color the history books.

43. According to the article, the Lusitania is a?


A. Airplane
B. Torpedo Boat
C. Ship
D. Train

44. In this statement from the article, “He was touted to be a middle-aged
fireman in the engine room.” Touted means?
A. Publicized
B. Demeaned
C. Set-up
D. Sighted

45. This passage sheds doubt on


A. The fact that three ships sank.
B. The thought that America was at war with other countries.
C. Frank Towers causing the ships to sink.
D. Urban folk tales are not always based on real people.

46. The title of this article could be


A. “Frank Towers, a Man of Mystery”
B. “Surviving the Impossible”
C. “The Legend of Frank Towers Debunked”
D. “How to Survive Doomed Ships”

47. According to this passage, which of the following phrases best captures the
intent of the article?
A. A stitch in time saves nine.
B. Don’t believe everything you read.
C. Seeing is believing.
D. There are many layers of an onion.

48. The article states that urban folk tales are created because
A. They are fun to hear.
B. Humanity likes to see someone triumph over tragedy
22

C. They trick people into believing lies.


D. People do not remember details clearly.

Reading 4

Medical documentation standards vary between state and federal organizations.


Hospitals and insurance companies have policies that regulate medical
documentation and terminology. Health care professionals must adhere to these
standards to support the level of treatment and the care plan administered.
Medicare is the leader in determining what is required in medical documentation
to support delivery of services. The other major insurance companies look to
Medicare for leadership with medical documentation and medical terminology.

Medical documentation and terminology are used to allow a continuity of care


between health care professionals. This terminology is universal and is the
language of health care providers. Appropriate medical terminology allows a
health care practitioner to clearly identify a colleague’s opinion and determine
delivery of services. If an error is present in medical documentation and
terminology, the patient will suffer the consequences. For example, if a vascular
surgeon identifies the need for a below knee amputation (BKA) on the right lower
extremity and an orthopedic doctor performs an above knee amputation (AKA)
on the right lower extremity, a malpractice suit will most certainly follow.

Medicare also sets standards of care that require appropriate documentation for
reimbursement purposes. Inappropriate medical documentation and terminology
does not support the proper delivery of services and payment to the hospital or
clinic will not be forth coming. Consequently, hospitals will go broke if multiple
claims are denied, because of inappropriate medical documentation and
terminology.

49. The main idea of this passage is?


A. The importance of proper medical documentation and terminology.
B. Documentation standards are universal.
C. Health care professionals are allowed to document however they want.
D. Medical documentation does not effect reimbursement.

50. Considering the information in this passage, you might suspect that hospitals
would teach their employees
A. How to maximize reimbursement with their documentation skills.
B. How to document pre-operatively.
C. A standardized set of medical abbreviations.
D. All of the above

51. According to the article, what could be the end result of poor documentation?
A. Claims denied.
23

B. Patients injured.
C. Hospital goes out of business.
D. All of the above.

52. What organization sets the standards for medical documentation according to
this article?
A. The medical profession
B. The healthcare professionals
C. Medicare
D. The hospital

53. The article states that health care professionals must adhere to these
standards. Adhere means?
A. Obtain
B. Comply
C. Include
D. Remember

54. Readers of this passage might expect the author to proceed to discuss
A. Other benefits of proper medical documentation
B. How to document properly
C. The relationship between medical terminology and documentation
D. Programs to implement for better documentation in hospitals

Reading 5

When Title IX came into effect in 1972, it challenged colleges to offer increased
sport opportunities for women. Since 1972, the number of women participating in
college sports has increased five fold. Colleges were required to offer improved
ratios of scholarships for women sports and to obtain sex equality in sports.

Generally, colleges compare the ratio of women in athletics to the ratio of women
in their undergraduate programs to determine compliance with Title IX. The
(CHE) Chronicle of Higher Education report in July 2002, stated that 45 colleges
are in compliance with Title IX. Thousands of colleges still have challenges to
become compliant with Title IX according to the CHE. Ohio State, Syracuse and
Texas Tech are three of the schools in compliance with Title IX according to the
CHE.

Many colleges have begun cutting back on the opportunities for men sports and
increasing the opportunities for women to be compliant with these new
guidelines. Sports such as wresting and gymnastics for men are being cut at
colleges, and sports such as golf and rowing are being offered to women.
Colleges have a set amount of money to dedicate to athletics and smaller men’s
sports are being cut to allow sports like football and baseball to continue.
24

College women have been offered new chances for scholarships and funding for
college. Many colleges are desperately seeking women for rowing, soccer, and
golf teams to reach equality ratios. College females, who need extra money for
college and have even a small amount of athletic ability, will be considered for
these positions. If a high school senior has kept in shape and likes working out,
then rowing in college may be close to a free ride. If a high school senior has
played golf several times, they may qualify for women’s college team at a small
school.

Currently, some male college athletes are crying foul play and charging
discrimination, if their sport program is being cut. However, colleges are trying to
balance the books and comply with Title IX. Changes are probably coming to
Title IX; however, the precedent of increased women participation in sports has
already been determined.

55. What is the main idea of this article?


A. How female students can get free money.
B. Discrimination of male athletes.
C. How increasing compliance with Title IX by colleges affected males and
females.
D. Changes to Title IX.

56. Which of the following statements are supported by the article?


A. Men’s sports are being completely eliminated for women’s sports.
B. Mediocre women athletes may be able to win scholarships at small
schools.
C. All colleges are compliant with Title IX.
D. Men’s Gymnastics is not a sport that is being cut by some schools for Title
IX.

57. Title IX is best described as


A. An act that encouraged schools to increase funding of female sports
determined by the ratio of women enrollees to the number of sports
opportunities.
B. An act that eliminates some male sports for women’s sports.
C. An act that is being followed by the majority of universities in the United
States.
D. An act that dictates that schools must be compliant with the standards
given by the CHE for college sports equality.

58. According to the article which of the following schools was not mentioned as
being compliant with Title IX.
A. Texas Tech
B. Syracuse
C. Tennessee State
25

D. Ohio State

59. According to the article, participation in women’s sports has increased how
much since Title IX was implemented.
A. 3 times
B. five fold
C. twice as much
D. one fifth as much

60. The article states that some male college athletes are crying foul play. In this
statement, foul play means
A. Bird
B. Unfair
C. Necessary
D. Fossick
26

Answer Key for Word Knowledge Section 2 (Practice Test 1)


1. B
2. D
3. C
4. B
5. A
6. D
7. B
8. C
9. A
10. D
11. A
12. D
13. C
14. B
15. D
16. A
17. C
18. B
19. A
20. D
21. A
22. A
23. C
24. A
25. D
26. B
27. A
28. D
29. B
30. A
27

Answer Key for Reading Comprehension Section 2 (Practice Test 1)

31. D
32. A
33. A
34. C
35. B
36. B
37. A
38. D
39. C
40. D
41. D
42. A
43. C
44. A
45. D
46. C
47. B
48. B
49. A
50. D
51. D
52. C
53. B
54. A
55. C
56. B
57. A
58. C
59. B
60. B
28

Section 3 Science (Practice Test 1)

Answer each question carefully using your science knowledge. The


correct answers are found after the section. You are allowed 60 minutes to
complete this test.

1. Which of the following cannot be considered a major tissue type?


A. Nerve tissue
B. Muscle tissue
C. Connective tissue
D. Extracellular tissue

2. Which of the following atoms is denoted by the symbol K?


A. Potassium
B. Calcium
C. Sodium
D. Helium

3. Which of the following represents the chemical formula for Ammonia?


A. NH4
B. NH3
C. NH2
D. NH1

4. Ions that have a net negative charge are called?


A. Anions
B. Cations
C. Molecules
D. Protons

5. Which of the following atoms is the most numerous in the body?


A. Chlorine
B. Nitrogen
C. Hydrogen
D. Calcium

6. Which of the following is the correct formula for pH?


A. pH = -log[H+]
B. pH = +log[H+]
C. pH = 10/-log[H+]
D. pH = 10/-log[H+]

7. Using percent of body weight calculations, which of the following categories is


found in the highest concentration in the body?
A. Nucleic acids
B. Lipids
29

C. Carbohydrates
D. Proteins

8. The three dimensional shape of a molecule is known as __________.


A. Conformation
B. Association
C. Alignment
D. Coagulation

9. Which of the following bases is not associated with deoxyribonucleic acid


(DNA)?
A. Adenine
B. Thymine
C. Uracil
D. Cytosine

10. Which of the following bases is not associated with ribonucleic acid (RNA)?
A. Adenine
B. Thymine
C. Uracil
D. Guanine

11. The atomic weight of an atom is the ratio of an atom’s mass compared to
____ atom?
A. Hydrogen
B. Carbon
C. Helium
D. Potassium

12. What is the characteristic number of covalent bonds that carbon forms?
A. 1
B. 2
C. 3
D. 4

13. DNA (Deoxyribonucleic acid) consists of ___ chains of nucleotides coiled


together.
A. 1
B. 2
C. 3
D. 4

14. Alkaline solutions have a pH that falls in which of the following ranges?
A. < 7.0
B. >7.0
C. <7.5
30

D. >5

15. Which of the following is not considered a major class of biological units?
A: Viruses
B: Eukaryotic cells
C: Prokaryotic cells
D: Cytosol cells

16. Which of the following is not considered a function of the plasma membrane
in cells?
A. Connecting cells together with junctions
B. Determining if chemical messengers are present
C. Manufacture proteins
D. Regulate flow of nutrients into and out of cell

17. Which of the following cellular organelles is considered the protein


manufacturing center of a cell?
A. Ribosomes
B. Endoplasmic Reticulum
C. Golgi Apparatus
D. Nucleus

18. One ____ is the heat required to increase the temperature of water 1 degree
if 1 gram of water is present on the Celsius scale.
A. Kilocalorie
B. Calorie
C. Joule
D. Ampere

19. Which of the following reactions most appropriate matches the following
definition: at a state of chemical equilibrium, the product of the reaction has
required almost all of the reactants.
A. Irreversible reaction
B. Reversible reaction
C. Combination reaction
D. Catalyst reaction

20. In humans, the primary molecule of energy transfer is considered?


A. ATP
B. ADP
C. CP
D. K

21. In which of the following stages of glucose breakdown is the most energy
produced?
A. Glycolysis
31

B. Krebs Cycle
C. Oxidative Phosphorylation
D. Citric Acid Cycle

22. Arterioles, venules, and capillaries are collectively known as _____.


A. Macrocirculation
B. Microcirculation
C. Pulmonary trunk
D. Hydrostatic gradient

23. Which of the following systems carries blood that is low on oxygen to the
lungs?
A. Pulmonary arteries
B. Pulmonary veins
C. Systemic system
D. Systemic arteries

24. Blood entering the right ventricle in the heart during normal circulation goes to
the ____ next.
A. Right atrium
B. Left atrium
C. Left ventricle
D. Lungs

25. Blood entering the left ventricle in the heart during normal circulation goes to
the ____ next.
A. Lungs
B. Systemic circulation
C. Right ventricle
D. Right atrium

26. Which of the following is not a function of the kidneys?


A. Assists in the regulation of blood pressure
B. Gluconeogenesis
C. Regulation of water balance in the body
D. Secreting enzymes

27. Which of the following tissue structures is not considered part of the urinary
system?
A. Ureter
B. Urethra
C. Bladder
D. Pancreas

28. Which of the following tissue structures is not considered part of the
gastrointestinal system?
32

A. Kidney
B. Liver
C. Gallbladder
D. Pancreas

29. The point in a female’s menstrual cycle that is most correlated with pregnancy
is?
A: Secretory phase
B: Proliferative phase
C: Ovulation
D: Follicular ovarian phase

30. Plasma cells make _____ in the body.


A. Antibodies
B. Plasma
C. Renin
D. Insulin

31. Which of the following symbols indicates force?


A. W
B. C
C. J
D. N

32. Which of the following symbols indicates energy?


A. N
B. C
C. J
D. M

33. Which of the following symbols indicates atomic number?


A. M
B. Z
C. C
D. E

34. Which of the following correlates with the following symbol on the periodic
table (Sn)?
A. Strontium
B. Silicon
C. Tin
D. Tungsten

35. Which of the following correlates with the following symbol on the periodic
table (Cr)?
A. Cesium
33

B. Copper
C. Chromium
D. Chlorine

36. Which of the following correlates with the following symbol on the periodic
table (Ti)?
A: Tin
B: Terbium
C: Thorium
D: Titanium

37. Which of the following has an atom number of 1?


A: Hydrogen
B: Oxygen
C: Carbon
D: Nitrogen

38. One mole of something is equal to ______.


A: Faraday’s Constant
B: Boltzmann’s Number
C: Planck’s Constant
D: Avagadro’s Number

39. The “sensory fingers” of a nerve are considered the _______.


A. Axons
B. Dendrites
C. Collateral extensions
D. Schwann cells

40. The central nervous system is composed of the ______ and ______.
A. Brain and peripheral nerves
B. Brain and spinal cord
C. Brain and afferent nerves
D. Brain and lower motor neurons

41. An atom that only has one electron in the outer orbit is considered a/an
_______.
A. Free radical
B. Ion
C. Isotope
D. Molecule

42. Solutes dissolve in a _____ to form a solution.


A. Oil based concentration
B. Water based concentration
34

C. Solvent
D. Gas

43. Glucose is considered a ____ sugar.


A. Simple
B. Double
C. Triple
D. Complex

44. Which of the following atoms is not found in carbohydrates?


A. Hydrogen
B. Oxygen
C. Carbon
D. Cesium

45. Which of the following is not considered a steroid?


A. Testosterone
B. Glycogen
C. Cholesterol
D. Estrogen

46. The sugar in each nucleotide of RNA is called?


A. Ribose
B. Deoxyribose
C. Renin
D. Ribosome

47. Which of the following cell structures conducts the nerve signal from the cell
body to target cells in neural tissue?
A. Dendrite
B. Axon
C. Nucleus
D. Nucleotides

48. The rhythmical changes in the body during a 24 hour period are known as?
A. Circadian rhythm
B. Circular cycle
C. Circadian entrainment
D. Potassium flow cycle

49. The movement of molecules as a result of random thermal motion across a


membrane is known as?
A. Net flux
B. Osmosis
C. Diffusion
D. Mediated transport
35

50. The process of evolution applies the principle of _____, to allow for survival of
the fittest.
A. Macro-evolution
B. Micro-evolution
C. Natural selection
D. Dominant selection

51. The human body has ___ pairs of chromosomes.


A. 15
B. 18
C. 21
D. 23

52. The process of converting DNA into RNA is known as?


A. Transcription
B. Translation
C. Transmutation
D. Tracking

53. Glucose is converted into the most ATP’s under which of the following
conditions?
A. Anaerobic
B. Aerobic
C. Moderate substrate
D. Enzymatic

54. The breakdown of organic molecules is known as?


A. Anabolism
B. Catabolism
C. Modulation
D. Coagulation

55. One calorie is the amount of heat necessary to raise ___ g of water ____ ° on
the Celsius scale.
A. 1,1
B. 1,2
C. 1,3
D. 1,5

56. The most ATP’s are produced in which of the following cell organelles?
A. Golgi apparatus
B. Endoplasmic reticulum
C. Cytosol
D. Mitochondria
36

57. What is the thin layer of tissue that lines the back of the eyeball that the
image of an object is projected upon?
A. Cornea
B. Lens
C. Optic nerve
D. Retina

58. Which of the following is not a small bone in the middle ear?
A. Incus
B. Stapes
C. Capitate
D. Malleus

59. The ___ and ___ provide information to the body about the body’s position in
space with movement from the ear.
A. Utricle and Hamate
B. Saccule and Hamate
C. Utricle and Saccule
D. Saccule and Pisiform

60. The olfactory nerve is cranial nerve ___.


A. I
B. II
C. III
D. IV
37

Answer Key for Science Section 3 (Practice Test 1)

1. D
2. A
3. B
4. A
5. C
6. A
7. D
8. A
9. C
10. B
11. B
12. D
13. B
14. B
15. D
16. C
17. A
18. B
19. A
20. A
21. C
22. B
23. A
24. D
25. B
26. D
27. D
28. A
29. C
30. A
31. D
32. C
33. B
34. C
35. C
36. D
37. A
38. D
39. B
40. B
41. A
42. C
43. A
44. D
38

45. B
46. A
47. B
48. A
49. C
50. C
51. D
52. A
53. B
54. B
55. A
56. D
57. D
58. C
59. C
60. A
39

Section 1 Mathematics (Practice Test 2)

1. 75% of 500
A. 365
B. 375
C. 387
D. 390

2. 45% of 600
A. 250
B. 260
C. 270
D. 280

3. (7 x 5) + (8 x 2) =
A. 51
B. 57
C. 85
D. 560

4. (8 ÷ 2) (12 ÷ 3) =
A. 1
B. 8
C. 12
D. 16

5. Which of the following numbers is a prime number?


A. 12
B. 25
C. 27
D. 31

6. Which number is a factor of 36?


A. 5
B. 7
C. 8
D. 9

7. 75 x 34 =
A. 1200
B. 2050
C. 2550
D. 3100

8. x + 372 = 853, x =
A. 455
40

B. 481
C. 520
D. 635

9. Convert .25 into fraction form.


A. ¼
B. ½
C. 1/8
D. 2/3

10. 60 grains are equal to 1 dram. How many grains are in 15 drams?
A. 900
B. 1020
C. 1220
D. 1300

11. A pitcher holds 7 ½ cups water. How many cups will 5 pitchers hold?
A. 34 ¼
B. 35 ½
C. 37 ½
D. 38 ¼

12. If a = 3, b= 4, c=5, then (a + b + c) 2 + (a – b - c) =


A. 124
B. 136
C. 138
D. 150

13. 0.85 =
A. 13/15
B. 17/20
C. 18/19
D. 19/22

14. Which fraction is closest to 2/3 without going over?


A. 6/13
B. 7/12
C. 11/16
D. 9/12

15. A puddle of water contained 72 pints of water. A rainstorm added 21% more
water to the puddle. Approximately, how many pints of water are now in the
puddle?
A. 76
B. 87
C. 92
41

D. 112

16. If x = 2 then x4 (x + 3) =
A. 72
B. 80
C. 96
D. 114

17. A circle graph is used to show the percent of patient types that a hospital
sees. How many degrees of the circle should the graph show if 1/3 of the
patient type is pediatric?
A. 90 degrees
B. 120 degrees
C. 220 degrees
D. 360 degrees

18. A traveler on vacation spent $ 25 at the grocery store the first week of school;
the next two weeks he spent $ 52; and the last week he spent $34. What was
his average food expenditure while he was on vacation?
A. $ 37.00
B. $ 38.25
C. $ 40.75
D. $ 52.00

19. 437.65 – 325.752 =


A. 111.898
B. 121.758
C. 122.348
D. 133.053

20. 43.3 x 23.03 =


A. 997.199
B. 999.999
C. 1010.03
D. 1111.01

21. How many nonoverlapping 2-inch x 2-inch squares are contained in an 8-inch
x 24- inch rectangle?
A. 32
B. 44
C. 48
D. 52

22. After going on diet for two weeks, you have lost 6% of you weight. Your
original weight was 157 lbs. What do you weigh now?
A. 132 lbs
B. 135.48 lbs
42

C. 144.98 lbs
D. 147.58 lbs

23. In order for a school to allow a vending machine to be placed next to the
cafeteria, 65% of the school’s population must ask for it. If 340 of the school’s
650 students have requested the vending machines, how many more are
needed to get the vending machines?
A. 75
B. 83
C. 89
D. 99

24. After purchasing a book that has a no return policy, the book goes on sale at
the bookstore for 15% less. You realize that you spent an extra $12.75 on the
book. What amount did you pay for the book originally?
A. $65
B. $75
C. $85
D. $95

25. Which of the following fractions have the largest value?


A. 8/15
B. 7/12
C. 6/13
D. 9/16

26. Round this number to the nearest hundredths 390.24657


A. 400
B. 390.247
C. 390.25
D. 390.2

27. To get 1 as an answer, you must multiply 4/5 by


A. 5/4
B. ½
C. 1
D. ¼

28. z = 4, z + 6 – (z+4)=
A. 2
B. 4
C. 6
D. 8
43

29. While working, patient’s sodium intake was 300 mg on Monday, 1240 mg on
Tuesday, 900 mg on Wednesday and Friday, and 1500 on Thursday. What
was the average intake of sodium while the patient was at work?
A. 476 mg
B. 754 mg
C. 968 mg
D. 998 mg

30. Which of the following numbers is correctly rounded to the nearest tenth?
A. 3.756 rounds to 3.76
B. 4.567 rounds to 4.5
C. 6.982 rounds to 7.0
D. 54.32 rounds to 54.4

31. 4.2% of 328 =


A. 12.7
B. 13.8
C. 14.2
D. 15.5

32. Which of the following fraction equal 0.625


A. 3/4
B. 5/6
C. 5/8
D. 2/3

33. Which of the following illustrates the distributive property of multiplication?


A. 2x + 5(z – 3) = 10x + 5z -15
B. 2x + 5(z – 3) = 2x + 5z - 3
C. 2x + 5(z – 3) = 2x + 5z - 15
D. 2x + 5(z – 3) = 2z + z – 15

34. Which of the following is the square root of 80


A. 4√5
B. 8
C. 5√4
E. 16

Questions 35 to 38 refer to the following information:


A school has a 50 x 60 yard rectangular playground. There are 3 classes
playing on it. 15 students are from Mrs. Red’s class, 12 are from Miss White’s
class, and 17 are from Ms. Brown’s class.

35. How many square yards does the playground cover?


A. 110
B. 300
44

C. 3,000
D. 30,000

36. How many students are playing on the playground?


A. 15
B. 22
C. 36
D. 44

37. If both sides of the playground are increased by 10%, what would the area be
in square yards?
A. 121
B. 1210
C. 3,000
D. 3,630

38. Ms. Brown’s class raised $400 to help put a fence around the playground.
The fence cost $15 a yard. How much more money per student would Ms.
Brown’s class have to raise to completely fence in the playground?
A. $165.12
B. $170.59
C. $183.57
D. $220.25

39. 160%=
A. 5/6
B. 6/5
C. 8/5
D. 9/6

40. A solution contains 6% calcium. How many milliliters of solution can be made
from 50 ml of calcium?
A. 830
B. 952
C. 1054
D. 2000
45

Answer Key for Mathematics Section 1 (Practice test 2)

1. B (500 x .75) = 375


2. C (600 x .45) = 270
3. A (35 +16 =51)
4. D (4 x 4 = 16)
5. D (31 is a prime number; it is only divisible by itself and one)
6. D (A factor is a number that divides evenly into another number.)
7. C (75 x 34 = 2550)
8. B (853-372= 481)
9. A
10. A (60 x 15 = 900)
11. C (15/2) (10/2) = 150/4 = 37.5
12. C (144) + (-6)= 138
13. B (85/100) reduces to 17/20
14. B Divide all fractions into decimals and compare
15. B (72 x .21= 15.12) 15.12+ 72= 87.12≈ 87pints
16. B (2 x 2 x 2 x 2) (5) = 16 x 5 = 80
17. B (360 degrees ÷ 3 = 120 degrees)
18. C ($25 + $52 + $52 + $34= $163) $163 ÷ 4 = $40.75
19. A ( 437.65-325.752 = 111.898)
20. A (43.3 x 23.03 = 997.199)
21. C (2 x 2= 4) , (8 x 24 = 192) 192 in2 ÷ 4 in2 = 48
22. D (157 lbs x .06 = 9.42 lbs) 157lbs- 9.42 lbs =147.58lbs
23. B (650 students x .65 = 422.5) 422.5 – 340 = 82.5 ≈ 83 more students
24. C (15/100 = 12.75/x) 15x= (100) (12.75) x= 1275/15 = $85
25. B (Divide out all fractions and compare decimal equivalents)
26. C (the hundredths place is two right of the decimal point)
27. A (to get one multiple any fraction by its reciprocal)
28. A (4 + 6 – (4 + 4) = 2 ( work inside of the parentheses first
29. C (300mg + 120 mg + 900mg + 900mg + 1500mg = 4840mg, 4840mg ÷ 5 =
968 mg
30. C (6.982 rounds to 7.0) look at the 8 it rounds the 9 to a 10 which adds a one
to the ones place, the 0 holds the tenths place
31. B (.042 x 328 = 13.8)
32. C ( change into fraction form 625/1000 then reduce)
33. C (multiply everything within the parentheses only)
34. Break 80 down into (16) (5), 16 has a square root of 4 leaving 5
35. C ( 50 x 60 = 3000)
36. D (15 + 12 + 17= 44)
37. D (50 x .1 = 5), 50+5=55, (60 x .1 = 6), 60 + 6 = 66, 66 x 55 = 3630 square
yards
38. B (60 + 60 + 50 + 50 = 220), (220 yards x $15/ yard = $3,300) (3,300 – 400=
2,900 ), (2,900 ÷17 = $170.59)
46

39. C ( 8/5 = 1.6) 1.6 x 100 = 160%


40. A (set up a ratio of 6/100 = 50/x) then solve by cross multiplying
47

Section 2 Verbal (Practice Test 2)

Word Knowledge: Read each sentence carefully. Then, on the basis of what
is stated in the sentence, select the correct completion of the incomplete
statement. The correct answers will be found at the end of this section.

1. Susan’s abhorrence of darkness prevents her from leaving her house at night.
Abhorrence means
A. Rationale
B. Hatred
C. Tremor
D. Belief

2. The girl displayed distraught behavior when she found out her puppy was
injured.
Distraught means.
A. Reckless
B. Shifty
C. Distressed
D. Unreasonable

3. The French exchange student spoke English as if it were her first language.
She was
A. Dandy
B. Fluent
C. Caustic
D. Talented

4. The prescription plan did not cover name brand drugs if there was a _______
substitute available.
A. Generic
B. Reasonable
C. Compatible
D. Complete

5. The somber crowd mourned the loss of their leader.


Somber means
A. Angry
B. Bitter
C. Melancholy
D. Excited

6. At age 65, the CEO of the company was retiring. He felt he had reached the
acme of his profession.
Acme means
A. Highest point
48

B. End
C. Bottom
D. Entrance

7. The genteel southern girl was known for her behavior.


Genteel means
A. Refined
B. Ambiguous
C. Smug
D. Loathsome

8. The mother attempted to mollify her son with toys.


Mollify means
A. Teach
B. Threatening
C. Soothe
D. Distract

9. The car accident caused a sliver of glass to cut the passenger’s optic nerve.
The passenger lost his
A. Arm
B. Movement
C. Smell
D. Vision

10. Some people accused John of thinking too much. He would sometimes
ponder on a subject for months at a time.
Ponder means
A. Resolve
B. Meditate
C. Discuss
D. Fret

11. The young artist had an unbridled passion for watercolors.


Unbridled means
A. Unrestrained
B. Unequaled
C. Underachieved
D. Distressed

12. The zephyr kept the students cool while they sat outside studying.
Zephyr means
A. Cloud
B. Tree
C. Shade
D. Wind
49

13. The pianist played his rendition of a sonata.


Sonata means
A. Instrumental composition
B. Piano
C. Play
D. Vocal score

14. The entertainer had no qualms about performing in front of two thousand
screaming fans.
Qualms means
A. Excitement
B. Illusions
C. Misgivings
D. Solace

15. The yearling still enjoyed being around its mother but was acting more
independent each day.
Yearling means
A. Student
B. Young animal
C. Type of fish
D. Bird

16. The financial planner had reach the top of his career; he felt he was at his
A. Performance
B. Stress level
C. Limit
D. Zenith

17. The siblings found solace in each other as they remembered the good times
with their father.
Solace means
A. Comfort
B. Despair
C. Dreams
D. Ambitions

18. The young boy sat passively as the principal yelled at him.
Passively means
A. Tiredly
B. Hungrily
C. Unresponsively
D. Readily

19. The teenage was accused of killing his father and mother. He was accused of
A. Sobriety
B. Misguidance
50

C. Misgamy
D. Parricide

20. Brian secrets to studying success relied on a system designed to assist with
the recollection of terms. His secret was the use of
A. Syllables
B. Memorabilia
C. Mnemonics
D. Puzzles

21. The bachelor never married. Most people thought it was because of
misogyny.
A. Fear of marriage
B. Hatred of women
C. Fear of People
D. Hatred of commitment

22. The intricacy of the mathematical equation drove the student crazy trying to
solve it.
Intricacy means
A. Activity
B. Ambiguity
C. Sections
D. Perplexity

23. The hybrid corn was immune to most common diseases.


Hybrid means
A. Cross-bred
B. Improved
C. Aged
D. Adjusted

24. The professor was humiliated when his students reported him to the dean for
verbal abuse.
Humiliated means
A. Scared
B. Shamed
C. Stunned
D. Surprised

25. The con artist hoodwinked the old lady when he sold her fraudulent
insurance.
Hoodwinked means
A. Affected
B. Criticized
C. Capitulated
51

D. Deceived

26. The movie star was accused of a misdemeanor when she stole 15 dollars
worth of merchandise from the store.
Misdemeanor means
A. Azide
B. Crime
C. Deed
D. Epoch

27. A false sense of intimacy was fostered by the serial killer with his victims.
Intimacy means
A. Security
B. Vitality
C. Deception
D. Familiarity

28. The bully pummeled the little boy until his older sister punched the bully.
Pummeled means
A. Battered
B. Chased
C. Tripped
D. Threatened

29. Most pagan religions believe that there is more than one god. These religions
are considered
A. Generous
B. Monotheistic
C. Polytheistic
D. Queer

30. John hesitated before choosing door number two.


Hesitated means
A. Danced
B. Filtered
C. Jumped
D. Paused
52

Section 2 Reading Comprehension (Practice Test 2 )

Reading 1

The SOAP format is used in most medical schools and allied health schools.
SOAP stands for Subjective, Objective, Assessment and Plan. Each of these
sections requires the basic understanding of medical terminology to allow
continuity of care. The SOAP format requires medical terminology that is
considered appropriate by the facility where you are working. Each facility
should have a listed set of appropriate medical abbreviations and surgical
abbreviations. The “S” includes the patient’s goal, patient’s pain complaint,
medical history and social history. The “O” includes the clinician data collection of
strength, range of motion, skin integrity and organ system functions. The “A”
includes the clinician’s opinion about the presented case, prognosis, diagnosis
and goals. The “P” includes the plan to progress to the goals set in the “A” and
the interventions that are necessary.

Some schools and are trying to get away from the SOAP format and going to a
problem based documentation plan. Basically, under this new plan only changes
in status need to be documentation with problem based documentation, other
than vitals sign and critical medical information. The problem with the SOAP
format is that is unclear when a clinician should stop documenting. As a result,
SOAP notes can turn into complete medical reassessments with a single
treatment.

A clinician’s documentation standards should be set high. Too often, medical


terminology and documentation are ambiguous. Clinicians often use arrows,
their own abbreviations and paraphrasing that is not professional. Paraphrasing
and inappropriate use of medical terminology can result in the firing of a clinician
with strong patient care skills. Hospitals and clinics are a business. If they are
losing money over poor documentation, changes will be made. Medicare has
made information available about frequent medical documentation errors.
Moreover, hospitals are trying to bring staff up to speed on these issues.
Overuse of abbreviations can lead to sentences that are unclear; therefore,
medical errors may have increased occurrence.

A medical professional will not be able to remember the definition of every


medical term. Medical terminology could fill volumes of dictionaries. However, a
medical professional can learn the prefixes, suffixes and word roots that allow for
an understanding of most medical terms.

31. Which of the following could be a title of this article?


A. “SOAP Notes Exposed”
B. “Medical documentation Condensed”
53

C. “The relationship of Medical Terminology and SOAP NOTES”


D. “SOAP Notes and the Palliative Implications”

32. The “O” in the SOAP note stands for.


A. Operational
B. Objective
C. Organization
D. Operations

33. Which of the following is considered a problem with SOAP notes?


A. When to document.
B. When to stop documenting.
C. When to abbreviate with medical terminology.
D. When to reiterate the main points

34. Which of the following statements is not supported in the above text?
A. SOAP notes require proper medical terminology.
B. SOAP notes can become complete reassessments of medical care.
C. Good clinicians have been fired over poor documentation.
D. Medical documentation must be reviewed by administration

35. According to the article, the effects of poor medical terminology use have not
resulted in which of the following?
A. Firing of clinicians
B. Medical errors
C. Poor documentation
D. SOAP note format changes

36. Future changes in medical documentation that may be inferred from this
article are:
A. Medical terminology will continue to change in the future.
B. Medical documentation errors will continue to increase in the future.
C. SOAP notes may require more structured text and format in the future.
D. SOAP notes will continue to require skilled clinicians.

Reading 2

Many scams of college freshman revolve around quick cash schemes. Some
involve working at home, mailing letters, and offering services to others that are
invisible. Some profitable quick cash schemes are selling plasma, typing papers,
providing transportation, and selling back books.

Various blood-related organizations will pay 30-60$ for platelets provided, if a


student can give platelets. Normally, giving platelets requires several hours
hooked up to IV tubing, and a minimal body weight. In addition, students with
54

certain rare blood conditions may be able to make profitable quick cash in just a
few hours.

Typing papers has always been the reliable college job through out the years.
Normally, students put up flyers in student study hot spots and wait for business.
Any rate can be charged, however if it isn’t reasonable, no one will call the listed
phone number. Last minute students often try and use this kind of service. The
typist may be asked to fill in missing phrases or transitional words in the provided
paper, but the typist should remember, who should be doing the work. Many
typists require at least half of the money up front and the other half would be paid
upon completion of the paper. Consequently, students that disappear off the
radar prior to payment will loose their initial investment. $10-15 an hour is a
reasonable price, but you may charge extra if you have a tight timeline.

If you have a car on campus, friends will probably ask for a ride frequently. If you
have a fuel-efficient car you may be able to make a few dollars by charging a flat
fee for transportation expenses. Limit the number of free rides around campus
and consider posting a small visible sign inside your vehicle requesting payment.
Be extremely wary of providing rides to strangers, and people that you don’t
really know. Criminal minds abound on campus so don’t become a victim, and
use sound judgement. Make sure your insurance is kept current in the event of
an accident.

Selling books back to the bookstore is another good way to make money. Books
in good condition will have more value upon return, than books in poor condition
with multiple entries in the margins. A student that has a little time to raise quick
cash may consider putting up flyers advertising books for resell. In general, a
student will make more money selling directly to another student, then selling
back to the bookstore. The bookstore may run out of used books quickly,
consequently the only available books to buy may be unused full price books.
Check out the bookstore to determine if this is the case. Be sure and post a flyer
close to the bookstore to pick up any potential customers going to the bookstore.

Be wary of schemes to work in Alaska, folding envelopes, and being a sales


agent on campus for a business with a P.O.Box address. Many students try
going to Alaska during the summer to work on fishing boats. However, summer
conditions off the coast of Alaska are not fun. Ten to twenty foot seas can be the
normal conditions of the day. A student would be required to work in all weather
conditions and operate on minimal amounts of sleep. Also many a student has
lost a finger, or had a traumatic injury working on fishing boats.

Folding envelopes is a slow tedious process that offers poor rewards on time and
return. Often money from the envelope’s company is slow in coming and short
on the investment of time. Businesses without a P.O. Box are often times
businesses without a permanent addresses and location. Scam artists
55

attempting to separate a college student from their money run some of these
businesses.

37. In this article the author focuses on:


A. The relationship between quick cash and college funding.
B. The relationship between quick cash and academic freedom
C. The relationship between college and scholarships.
D. The relationship between cost and college

38. Which of the following was not considered a scheme to make money in
college in the article?
A. Selling plasma
B. Selling back books
C. Typing papers
D. Working full-time

39. Which of the following was not considered a possible mechanism of action
used by scam artists?
A. Folding envelopes scheme
B. Questionable business addresses
C. Selling vacations
D. Alaska opportunities

40. The article implies that selling back books is:


A. A difficult and complicated process.
B. Harder if selling to underclassmen directly: however, it may be more
profitable.
C. An easy way to earn a small amount of money in a hurry.
D. Profitable

41. In this article, the author reports that typing papers is:
A. A difficult and complicated process
B. Always profitable
C. Harder if typing for upperclassmen.
D. Sometimes a last minute business.

42. According to the passage, boats in Alaska:


A. May operate in 10-20 foot seas
B. Offer comfortable quarters
C. Provide good medical care
D. Offer good food

Reading 3
56

Generally, an applicant must be hired by a police department and then be sent to


the police officer academy for further training. Consequently, the selection
process for the police officer academy occurs primarily with a specific police
department. Once you have met all of the necessary requirements and have
been hired, the police department that you work for will set up officer training.
Smaller police departments in rural settings do not run their own police officer
academies. They rely on regional academy for officer training. Most of these
regional academies are in urban areas.

Most of the time, urban police officers make more money than rural police
officers. The urban environment offers a large tax base than can support higher
salaries. These higher salary positions are often much more selective than rural
police because the rural police department may less applicants due to poorer
salaries. Higher salary positions in the urban police officer force may require
additional hurdles to becoming a patrol officer. Sometimes, police officers are
required to participate in the local corrections department, until a patrol officer
position opens up. Consequently, patrol officer positions may be at a premium.

Moreover, new graduates or junior officers are often assigned the major holidays
and patrols at night. These patrols are tough because of holiday activity, and the
criminal element likes the cover of darkness to operate under. Be prepared to be
distant from family and friends as a junior officer during holidays.

According the Bureau of Labor and Statistics, the 2000 median annual earnings
for police patrol officers was $39,790 nationwide. In 2000, median earnings were
broken down as follows:

State: $44,400
Local: $39,710
Federal: $37,760

According to the International City-County Management Association’s annual


Police and Fire Personnel, Salaries and Expenditures Survey in 2000 the
following data was concluded:

Min. Pay Max. Pay


Police Chief $62,640 $78,580
Deputy Chief $53,740 $67,370
Police Captain $51,580 $64,230
Police Lieutenant $47,750 $57,740
Police Sergeant $42,570 $50,670
Police Corporal $35,370 $43,830
Police Officer $31,410 $43,450

It should be noted that whenever a city or town increases police officer pay, there
is sometimes a surge in applicants. Consequently, a hiring freeze may occur
57

following a hiring surge. Potential police officers should decide quickly following
a pay increase if they are committed enough to join the police force.

43. The main idea of this passage is


A. Law Enforcement Training and Pay
B. The Criminal Element
C. Urban Police Officers
D. Hiring surges in Law Enforcement Jobs

44. According to the article, which profession would a Law Enforcement Agent
hold if they made 44,000 a year?
A. Police Officer
B. Police Corporal
C. Police Sergeant
A. Police Captain

45. According to the article, which of the following is a true statement?


A. Urban police officers make less than rural officers.
B. Most officers desire local correctional department jobs.
C. During holidays, most police departments have their most experience staff
on duty.
D. Federal police officers make less money than state police officers on
average.

46. According to the article, urban areas are able to offer higher pay because
A. of a larger tax base.
B. they require less police forces.
C. they pay their fire department less.
D. they have less expenditures.

47. Which of the following statements is supported by the article?


A. Once you become a police officer, you will not have a pay increase.
B. Most rural areas do not have their own training facilities.
C. Police Captains are the lowest paid of the law enforcement officials
D. Rural police officers make more money than urban police officers.

48. According to the article, a junior officer may work difficult shifts. They should
expect
A. better pay than senior officers.
B. to be away from their families at night and on holidays.
C. to be training rurally.
D. to be transferred to another shift.
58

Reading 4

Ask an interviewer what the number one attribute they are looking for in an
interviewee and you’ll invariably get the same response – Attitude. The most
important impression you want to leave your interviewer with is that you have the
right attitude for the school. Other characteristics are important, such as
intelligence and experience, but they aren’t as significant to your success in the
interview as your attitude.

Every potential applicant has their qualifications all polished up and displayed
proudly on their application. But those accomplishments are all in the past. An
interviewer has to look at you in the present to determine how successful they
think you will be in the future. The attitude you display is fundamental to their
perception of your future success. Almost every employer would rather have a
team player with a great attitude working at 100%, rather than a flashy superstar
working at 50%. Regardless of your usual personality, remember that you’re
only in the interview for 30 minutes on average. In that short amount of time, do
whatever it takes to wear a smile, remain positive, and exude a positive attitude.

It is easy to blame others for your mistakes or shortcomings. If an interviewer


asks a question about something that you may be embarrassed about, don’t
immediately become negative and blame other people or situations. Accept
responsibility for your actions and your past but make sure that anything negative
does not remain the focus. Turn negatives into positives. For example, suppose
an interviewer asks, “I see on your application that you show you dropped out of
high school. Could you explain what happened?”

“As a teenager, I made my share of mistakes. That time in my life was the best
thing that ever happened to me. I needed to grow-up, and I did. I learned to take
responsibility for my actions. I earned my GED and have since held the same
job for over two years. I have learned that I want to better myself, and I have
been extremely active in working with troubled youths. My own background has
offered an excellent background for this type of community service. I work to
discourage them from what appears to be the easy way out which in the end
turns out to be much harder.”

Whatever you do; do not constantly shift the blame to others. It is okay to have
had shortcomings in your past. The key in an interview is to show how you’ve
learned from those experiences and have moved on and overcame them.

49. The best title for this selection is


A. “How to Maximize Your Interview.”
B. “Turning Bad into Good.”
C. “Presenting a Polished Image.”
D. “Covering Your Mistakes.”
59

50. According to the article the number one most important attribute a person
needs to display in an interview is
A. arousing the interviewer’s interest
B. apologetic about mistakes
C. calmness
D. a positive attitude

51. According to the article, if you went to prison, you should


A. deny that it happened.
B. own up to your mistake and describe what you have learned from it.
C. explain how it was not your fault.
D. express your sorrow and say how you cannot get pass the experience.

52. If a teacher had to select among the following student applicants, which
would he select according to the article?
A. a super star working at 50%
B. a perfectionist
C. a hard worker with a positive attitude
D. an underachiever

53. Which of the following statements should you not make during an interview?
A. “If you give me a chance, I will prove to you what a positive asset I can be
to your program.”
B. “The community service that I had to do as a juvenile delinquent helped
me develop into the hard working person I am today.“
C. “I wouldn’t even have a record if my parents hadn’t turned me in.”
D. “I have overcome my drug problem; I learned so much about myself from
the personal development classes I attended in rehab.”

54. According to the article, all accomplishments that you have had so far in life
are
A. unimportant.
B. in the past.
C. determine your future.
D. shape what type of applicant you will be.

Reading 5

Fishing with live bait is not as popular as it used to be. The majority of bass
fishermen use artificial lures. Even with this being the case, the record book
show that live bait may be the way to go. The second largest bass weighed 21
pounds, 3 ounces and was taken on a crawfish. The third largest bass weighed
20 pounds, 15 ounces and was caught on a night crawler.
60

Different weather and water conditions affect how a bass reacts to live bait. Clear
water conditions make an artificial lure easy for a bass to identify. Largemouth
bass prefer live bait after a cold front or when their optimal feeding temperature is
above or below normal. Slow-moving bass prefer slow-moving bait. Most artificial
lures just do not look real when moving at a slow pace. Live bait on light line has
a very realistic and tempting movement especially when it is dragged slowly
along the bottom of the chosen body of water.

Another commonly used technique is adding live bait to a lure. This gives the lure
a natural movement and works well with jigs, spinners, or spinnerbaits. The most
often used bait for this technique is worms and minnows.

Largemouth bass will strike on a variety of live bait since they eat a wider variety
of native foods than most fish. Some of the most popular live bass bait includes
waterdogs, frogs, crawfish, shrimp, golden shiners, and night crawlers.

55. Which of the following would be an appropriate title for the article
A. Champion Fishing
B. Fishing for Bass with Live Bait
C. Waterdogs and How to Use Them
D. Popular Bass Bait

56. According to the article, live bait looks more realistic than artificial bait in
A. saltwater
B. freshwater
C. clear water
D. dark water

57. According to the article, which types of live bait work well when added to an
artificial lure?
A. waterdogs and frogs
B. shrimp and crawfish
C. minnows and worms
D. golden shiners and night crawlers

58. Which of the following is a true statement according to the article?


A. Two of the three largest bass were caught with live bait
B. Jigs and spinnerbaits do not function well with live bait.
C. Artificial bait looks as real as live bait when used at a slow pace.
D. The majority of fishermen use live bait.

59. When using live bait which type of line should you use
A. heavy weight
B. 40 lbs weight
C. medium weight
D. light weight
61

60. The article states when their optimal feeding temperature is above or below
normal. Optimal means
A. Perfect
B. Fastest
C. Slowest
D. Worst
62

Answer Key for Word Knowledge Section 2 (Practice Test 2)

1. B
2. C
3. B
4. A
5. C
6. A
7. A
8. C
9. D
10. B
11. A
12. D
13. A
14. C
15. B
16. D
17. A
18. C
19. D
20. C
21. B
22. D
23. A
24. B
25. D
26. B
27. D
28. A
29. C
30. D
63

Answer Key for Reading Comprehension Section 2 (Practice Test 2)

31. C
32. B
33. B
34. D
35. D
36. C
37. A
38. D
39. C
40. B
41. D
42. A
43. A
44. C
45. D
46. A
47. B
48. B
49. A
50. D
51. B
52. C
53. C
54. B
55. B
56. C
57. C
58. A
59. D
60. A
64

Section 3 Science (Practice Test 2)

Answer each problem carefully. The correct answers are found after the
section. You are allowed 60 minutes to complete this test.

1. The compound NaCl stands for:


A. Sodium Chloride
B. Sodium Chlorine
C. Niabium Chloride
D. Niabium Chlorine

2. Household ammonia and human blood are considered which of the following?
A. Acids
B. Bases
C. Salts
D. Electrolytes

3. Human blood is what percent water?


A. 55.4%
B. 68.3%
C. 84.9%
D. 91.5%

4. Which of the following terms matches the given definition? ______ is the
changing of substances into a different form that entered the body.
A. Absorption
B. Circulation
C. Assimilation
D. Responsiveness

5. When atoms of different elements combine which of the following is formed?


A. Compounds
B. Isotopes
C. Electrons
D. Molecules

6. Which of the following reaction terms correlates with the given chemical
reaction? AB + CD → AD + CB
A. Decomposition reaction
B. Synthesis reaction
C. Exchange reaction
D. Structural reaction

7. Which of the following reaction terms correlates with the given chemical
reaction? AB → A + B
65

A. Decomposition reaction
B. Synthesis reaction
C. Exchange reaction
D. Structural reaction

8. Which of the following is considered the firmest type of connective tissue?


A. Hyaline cartilage
B. Bone
C. Ligaments
D. Elastic cartilage

9. Which of the following is not a characteristic of smooth muscle?


A. Involuntary muscle contraction
B. Found in blood vessels
C. Cells lack striations
D. Presence of an intercalated disk

10. Which of the following appropriate indicates the correct sequence of mitosis?
A. Telophase, Anaphase, Metaphase, Prophase
B. Anaphase, Metaphase, Telophase, Prophase
C. Prophase, Metaphase, Telophase, Anaphase
D. Prophase, Metaphase, Anaphase, Telophase

11. At what point during Mitosis does the chromosome (chromatids) separate?
A. Metaphase
B. Anaphase
C. Telophase
D. Prophase

12. The process of a white blood cell (WBC) engulfing a bacterial cell is ______.
A. Phagocytosis
B. Osmosis
C. Transfusion
D. Active transport

13. What is considered the building blocks of protein?


A. Enzymes
B. Nucleic acids
C. Amino acids
D. Chromatids

14. Which of the following is not a characteristic of proteins?


A. Energy source
B. Function as hormones
C. Structural components
D. Breakdown fats
66

15. Lactose is considered a ________.


A. Simple sugar
B. Double sugar
C. Triple sugar
D. Expanded sugar

16. Is the following statement true or false? The increase in momentum by one
body is equal to the momentum decreased by the other body.
A. True
B. False

17. Is the following statement true or false? Momentum is the product of the
velocity and mass of an object.
A. True
B. False

18. Is the following statement true or false? Power is the rate at which work is
done.
A. True
B. False

19. Is the following statement true or false? Inertia is an object’s resistance to


change in motion.
A. True
B. False

20. Which of the following corresponds with the statement: An object will
continue in a state of rest, unless it is acted upon by an unbalanced force.
A. Newton’s third law of motion
B. Newton’s second law of motion
C. Newton’s first law of motion
D. Charles law

21. Homeostasis control of body functions is done through a system known as?
A. Positive feedback
B. Negative feedback
C. Stop controls
D. Transitional control

22. Different tissues make up ______.


A. Organelles
B. Organs
C. Macromolecules
D. Proteins
67

23. Which of the following is not considered an organ?


A. Skin
B. Femur
C. Kidney
D. Nerve cells

24. Which of the following is not considered part of the endocrine system?
A. Hormones
B. Parathyroid
C. Adrenal glands
D. Smooth muscle

25. All of the following are considered part of which organ system? (skin, sweat
glands, and nails)
A. Endocrine
B. Skeletal
C. Integumentary
D. Muscular

26. Atoms that gain or loose electrons are called ____.


A. Isotopes
B. Molecules
C. Ions
D. Neutrons

27. The atomic number of an element is classified by the number of _____ in the
atoms.
A. Protons
B. Neutrons
C. Ions
D. Electrons

28. How many electrons are found in the second shell of atom, if the atom is filled
to capacity in the second shell?
A. 2
B. 6
C. 8
D. 12

29. Elements are made of small particles known as?


A. Ions
B. Protons
C. Molecules
D. Atoms

30. The nucleus of an atom is made of ____ and _____.


68

A. Protons and Neutrons


B. Protons and Electrons
C. Neutrons and Electrons
D. Neutrons and Isotopes

31. How many electrons are found in the first shell of atom, if the atom is filled to
capacity in the first shell?
A. 2
B. 6
C. 8
D. 12

32. Which of the following correlates with the following symbol on the periodic
table (Au)?
A. Copper
B. Silver
C. Gold
D. Argon

33. Which of the following correlates with the following symbol on the periodic
table (Ag)?
A. Copper
B. Silver
C. Gold
D. Argon

34. Which of the following correlates with the following symbol on the periodic
table (Cm)?
A. Cesium
B. Curium
C. Calcium
D. Carbon

35. The majority of the periodic table is comprised of elements that are
considered?
A. Metals
B. Metalloids
C. Nonmetals
D. Transitional metals

36. The second law of Thermodynamics indicates: Entropy is conserved for a


reversible process.
A. True
B. False

37. The ability of unspecialized cells to turn into specialized cells is known as?
69

A. Organ differentiation
B. Cell differentiation
C. Transmutation
D. Mutation

38. Which of the following structures is not considered part of the respiratory
system?
A. Larynx
B. Bronchi
C. Trachea
D. Thyroid

39. What is the electric charge of an electron?


A. 1
B. 0
C. -1
D. –2

40. Which of the following is not considered a major element in the human body?
A. Carbon
B. Nitrogen
C. Oxygen
D. Iron

41. When skeletal muscle contracts which of the following mechanisms is


applied?
A. Contract/Relax mechanism
B. Sliding-filament mechanism
C. Hold/Contract/Relax mechanism
D. Mono-filament mechanism

42. Which of the following is a unit indicating electrical power?


A. Ampere
B. Watt
C. Coulomb
D. Joule

43. Caffine is considered a ______.


A. Dieuretic
B. Depressant
C. Drug
D. Food preservative

44. Lactose intolerant individuals do not tolerate _____.


A. Milk
B. Alcohol
70

C. Sulfa medications
D. Steak

45. What type of cells engulfs bacterial cells by a process of phagocytosis?


A. Staph cells
B. Plasma cells
C. Hemoglobin cells
D. White blood cells

46. Which are of the brain controls limb movement related to balance and vision
imput?
A. Frontol lobe
B. Parietal lobe
C. Temporal lobe
D. Occipital lobe

47. The specific scientific name of an organism is identified by _____ and ____.
A. Kingdom and Phylum
B. Order and Species
C. Genus and Species
D. Genus and Order

48. Compounds that are in various structure alignments, however are the same
mass and made of the same elements are called?
A. Isotopes
B. Isomers
C. Isotonic compounds
D. Isometric compounds

49. Insulin is produced by what type of cell in the pancreas?


A. Plasma cells
B. CD4 cells
C. Helper cells
D. Islet of Langerhan’s cells

50. The process of converting sunlight into energy in plants is known as?
A. Chlorosynthesis
B. Photosynthesis
C. Translation
D. Transcription

51. Gregor Mendel’s ground breaking genetic research was conducted on _____.
A. Peanuts
B. Bean plants
C. Pea plants
D. Roses
71

52. The thyroid gland requires ______ to function.


A. Calcium
B. Potassium
C. Iodine
D. Minerals

53. The process of photosynthesis requires _____ to work.


A. Oxygen
B. Carbon Dioxide
C. Nitrogen
D. Zinc

54. Which organ in the body is not a major control of blood pressure regulation?
A. Heart
B. Lungs
C. Kidneys
D. Skin

55. Which of the following bones is not part of the axial skeleton?
A. Vertebra
B. Rib
C. Clavicle
D. Tibia

56. Distance and force multiplied together indicate?


A. Power
B. Joules
C. Work
D. Energy

57. Which of the following organisms is considered an invertebrate?


A. Cat
B. Starfish
C. Eagle
D. Platypus

58. The process of cytoplasm division during plant cell replication is known as?
A. Cytokinesis
B. Photosynthesis
C. Cytosynthesis
D. Cytoplasmosis

59. Gas exchange occurs in which of following tissues?


A. Veins
B. Arteries
72

C. Capillaries
D. Skin

60. The end products of photosynthesis are ____ and _____.


A. Oxygen and Glucose
B. Glucose and Nitrogen
C. Nitrogen and Carbon Dioxide
D. Carbon Dioxide and Glucose
73

Answer Key for Science Section 3 (Practice Test 2)

1. A
2. B
3. D
4. C
5. A
6. C
7. A
8. B
9. D
10. D
11. B
12. A
13. C
14. D
15. B
16. A
17. A
18. A
19. A
20. C
21. B
22. B
23. D
24. D
25. C
26. C
27. A
28. C
29. D
30. A
31. A
32. C
33. B
34. B
35. A
36. A
37. B
38. D
39. C
40. D
41. B
42. B
43. C
44. A
45. D
74

46. D
47. C
48. B
49. D
50. B
51. C
52. C
53. B
54. D
55. D
56. C
57. B
58. A
59. C
60. A
75

Section 1 Mathematics (Practice Test 3)

1. 78 x 52
A. 3,066
B. 4,056
C. 4,283
D. 5,285

2. Stephanie is running a temperature of 103.7° F. The average human body


temperature is 98.7°F. What percent higher is Stephanie’s temperature than
normal?
A. 3%
B. 5%
C. 7%
D. 8%

3. If 60 grains are equal to one dram. How many grains are in 12 drams?
A. 500
B. 625
C. 720
D. 850

4. x + 230 = 542
A. 202
B. 212
C. 302
D. 312

5. 32.74 ÷ .03
A. 1.09
B. 10.91
C. 191.02
D. 1091.33

6. A nurse takes a patient’s pulse rate over 10 seconds and records 14. She
must then rank the pulse (beats per minute) on a scale. Bradycardia is below
40 bpm. Normal is between 40 and 120 bpm. Tachycardia is above 120 bpm.
What should she record the pulse as
A. Bradycardia
B. Normal
C. Tachycardia
D. None of the Above

7. If A>B and C>B, then the values of A, B, C could be


A. A=5, B=3, C=4
B. A=4, B=3, C=2
76

C. A=3, B=4, C=5


D. A=5, B=4, B=4

8. 6 x 5 x 7 x 1 =
A. 200
B. 210
C. 220
D. 230

9. 120.7502 rounded to the hundredths place


A. 120.750
B. 120.75
C. 120.8
D. 100

10. A vase holds 6 cups of water. How many cups of water will 6 vases hold?
A. 18 cups
B. 36 cups
C. 42 cups
D. 500 cups

11. Which of the following is a set of prime numbers


A. 7, 11
B. 21, 22
C. 5, 15
D. 2, 9

12. A parallelogram has two angles that are 75 degrees. What are the other two
angles?
A. 90 degrees
B. 105 degrees
C. 110 degrees
D. 120 degrees

13. An iron supplement contains 10 mg of iron. The average woman requires 18


mg of iron daily. How much more iron does a woman need to consume after
taking the supplement to meet her daily requirements?
A. 4 mg
B. 8 mg
C. 18 mg
D. 22 mg

14. Which of the following decimals equals 4.5%


A. .045
B. .45
C. 4.5
77

D. 45

15. Simplify: 4 – 5 (x + 3) =
A. 4 –5 x + 15
B. 4 – 5x – 3
C. 4 + 5x + 15
D. 4 – 5x – 15

16. If the replacement set for x is a set of positive integers, then the solution set
for x + 4 ≤ 7 is
A. 1,2
B. 1,2,3
C. 1,2,3,4
D. 1,2,3,4,5

17. Jamie has $ 550 in her savings account. She withdrew 15% of it. What is her
balance?
A. $82.50
B. $400.25
C. $467.50
D. $532.35

18. The cost of aspirin in a hospital increased from $1.56 a pill to $4.43 over the
past 20 years. What is the percent increase?
A. 25%
B. 28%
C. 257%
D. 284%

19. If t =7, then t3 + t4 =


A. 2654
B. 2744
C. 2935
D. 3010
20. Mr. Ruiz came into the emergency room at 9:15 PM. He did not go home until
10 hours and 30 minutes later. What time did he leave?
A. 6:30 AM
B. 6:45 AM
C. 7:45 AM
D. 8:15 AM

21. On an average day, the ratio of elective surgeries versus emergency


surgeries is 12:3. For every 36 elective surgeries, how many emergency
surgeries should be expected?
A. 9
B. 12
78

C. 15
D. 18

22. The toxic level of a specific drug is 1500 mg per day, and you have an order
to give a patient 370 mg dose of medication every 6 hours. How many
milligrams is the patient under the toxic level?
A. 20 mg
B. 200 mg
C. 370 mg
D. You have exceeded the toxic level.

23. The area of a triangle with a base of 13 cm and a height of 14 cm is


A. 27 cm
B. 91 cm
C. 350 cm
D. 182 cm

24. Round off 345.678 to the nearest tens place


A. 345.7
B. 345
C. 350
D. 300

25. How many liters are in 9,754 milliliters?


A. 9.754 liters
B. 97.54 liters
C. 975.4 liters
D. 9754 liters

26. Over the past week, George purchased some magazines and several
newspapers for $17.66. The magazines cost $2.13 each and newspapers
cost $ 0.55 each. How many magazines and newspapers did George
purchase?
A. 5 newspapers and 7 magazines
B. 4 newspapers and 5 magazines
C. 4 newspapers and 7 magazines
D. 7 newspapers and 4 magazines

27. A gambler lost $34 his first day in Las Vegas. The second he won $27. The
third day he lost $24 and the fourth day he gained $29. How much did he win
or lose during his four-day stay in Las Vegas?
A. He won $1.
B. He lost $2.
C. He lost $4.
D. He broke even.
79

28. What number is 43% of 657?


A. 265.31
B. 282.51
C. 325.31
D. 344.51

29. A regular octagon’s perimeter is 40 cm long. How long is each side?


A. 5
B. 7
C. 8
D. 10

30. A patient’s temperature increased from 98.7°F to 104.3°F over the course of 6
hours. What was the average degree change per hour?
A. 0.86°F/hr
B. 0.89°F/hr
C. 0.93°F/hr
D. 1.05°F/hr

31. If x < 7, then x could equal


A. 6
B. 7
C. 8
D. 9

32. 12% of the student population is expected to be exposed to the flu. Of the
12%, 45% will actually display symptoms of the infection. If the student
population is 450, then approximately how many will actually display
symptoms of the infection?
A. 12 students
B. 15 students
C. 24 students
D. 54 students

33. Jill has 3 times more nail polishes than her sister Jessica. Combined, they
have a total of 20 nail polishes. If n represents the number of nail polishes
that Jessica has, then which of the following equations depict the total
number of nail polishes?
A. 21 = 3n - n
B. 21 = 3n – 3n
C. 21 = n + n
D. 21 = 3n + n

34. If the daily cost of a hospital is represented by a circle graph, how many
degrees would be needed to show that the hospital spends 45% of its daily
cost on employee benefits and pay?
80

A. 120
B. 148
C. 162
D. 175

35. If 12% of a number is equal to 65.52, then the number is equal to


A. 225
B. 327
C. 521
D. 546

The following statement applies to numbers 36 through 40.

A storekeeper had to make 12% over cost in the first quarter of the year to stay in
business. In January his expenditures totaled $12,455. In February, they totaled
$13,365. In March and April, his expenses were $12,487 and $15,265,
respectively. His total revenue was $68,342.

36. What was the average expenditure of the storekeeper over the first four
months of the year?
A. $12,456
B. $13,393
C. $14,867
D. $15.263

37. How much profit did the storekeeper make?


A. $14,770
B. $15,432
C. $15,965
D. $16,220

38. What percent over cost did the storekeeper make?


A. 12.25%
B. 18.15%
C. 22.75%
D. 27.57%

39. In which month did he incur the least amount of expenditures?


A. January
B. February
C. March
D. April

40. Will the storekeeper be able to stay in business according to the statement
above?
A. Yes
81

B. No
C. There is not enough information to make decision.
D. None of the above

Answer Key for Mathematics Section 3 (Practice Test 3)

1. B (78 x 52) = 4056


2. B (103.5 – 98.7 = 5, 5/98.7 = .05, .05 x 100 = 5%)
3. C (1/60 x 12/x) = 720
4. D (542-230) = 312
5. D (32.74/.03) = 1091.33
6. B 84
7. A A=5, C=4, B=3
8. B 210
9. B 120.75
10. B (6 x 6 cups) = 36 cups
11. A Prime numbers are divisible by 1 and themselves only
12. B (360-75-75) = 105
13. B (18-10) = 8mg
14. C
15. D (4 – 5x –15)
16. B (1, 2, and 3 would all work)
17. C ($550 x .15 = 82.5, 550-82.5 = $467.50)
18. D ($4.43-$1.56= 2.84, 2.84 x 100 = 284%)
19. B (343 + 2401) = 2744
20. C
21. A (12/3 x 36/x) = 9
22. A
23. B (1/2bh)
24. C (350)
25. A (9754/1000)= 9.754
26. A
27. B (-34 + 27 + -24 + 29) = -2
28. B (657 x .43) = 282.51
29. A (40/8) = 5
30. C (104.3 –98.7) = 5.6, 5.6/6 = .93
31. A (6 < 7)
32. C (450 x .12
33. D (20 = 3n + n)
34. C (360 x .45) = 162
35. D
36. B
37. A (68342-53572 = 14770)
38. D (14770/53572 = 27.57)
39. A
82

40. A Yes, he made 27.57% above cost, and he only needed 12% to stay in
business.
83

Section 2 Verbal (Practice Test 3)

1. The prince abdicated the crown and returned to his castle.


Abdicated means
A. Gave up
B. Sold
C. Reinvested into
D. Auctioned

2. The convicted criminal absconded prior to the sentencing phase of the trial.
Absconded means
A. Touched the jury
B. Reported immediately
C. Left after discussion
D. Departed suddenly

3. The aural component of balance is critical for postural control during


ambulation.
Aural means
A. Vision
B. Hearing
C. Sight
D. Touch

4. The old man was benevolent with his fortune.


Benevolent means
A. Secretive
B. Stingy
C. Kind
D. Careful

5. The extra dirt was a key buttress to the foundation.


Buttress means
A. Limiting factor
B. Support
C. Overwhelming condition
D. Obstacle

6. The cathode of a battery was removed.


Cathode means
A. Positive pole
B. Negative pole
C. Neutral pole
D. Opposite pole

7. The doctor was known as a charlatan over the years of his practice.
Charlatan means
84

A. Quack
B. Knowledgeable physician
C. Procedural physician
D. Medical examiner

8. The wound exhibited signs of copious drainage requiring medical intervention.


Copious means
A. Minimal
B. Clear
C. Maximal
D. Foul

9. The attorney accused the witness of defaming the defendant.


Defaming means
A. Killing
B. Badgering
C. Suffocating
D. Slandering

10. The detective was able to derive the facts of the case.
Derive means
A. Desist
B. Deter
C. Devise
D. Deduce

11. The scientist was able to evoke powerful emotions from her audience.
Evoke means
A. Sell
B. Calm
C. Call forth
D. Exaggerate

12. The judge was fallible during deliberation.


Fallible means
A. Careful not to err.
B. Falsely accused
C. Loyal to his supporters
D. Capable of mistakes

13. The chemist collected the germane data during the experiment.
Germane means
A. Relevant
B. Obscure
C. Limited
D. Usual
85

14. The desperados held up in a grotto in New Mexico during the escape.
Grotto means
A. Large cave
B. Small cavern
C. Hotel
D. Motel

15. The official exhibited a heedless attitude when dealing with the dignitaries.
Heedless means
A. Thoughtless
B. Pleasant
C. Friendly
D. Bitter

16. The Sherman tank commander noted innumerable troops moving forward
against his position.
Innumerable means
A. Limited
B. Weary
C. Countless
D. Harmless

17. The general tried to instill in his troops the hope of victory.
Instill means
A. Infuse
B. Delay
C. Inscribe
D. Indict

18. The winning team of the World Series often has a jovial attitude.
Jovial means
A. Merry
B. Sad
C. Somber
D. Laborious

19. The plant entered the latent phase of development in the fall.
Latent means
A. First
B. Growth
C. Last
D. Dormant

20. The yacht club members were excited about conditions on the loch.
Loch means
86

A. Water
B. Lake
C. Gulf
D. Ocean

21. A lyre was played in ancient Rome.


Lyre means
A. Stringed instrument in the harp class
B. Percussion instrument
C. Wind instrument in the wind class
D. Rhythmical percussion device

22. The labyrinth caused confusion to the attacking troops.


Labyrinth means
A. Sound
B. Noise
C. Maze
D. Bulwarks

23. The wound was necrotic when examined.


Necrotic means
A. Healing
B. Dying tissue
C. Nauseating
D. Infinite

24. The defendant exhibited a peevish appearance.


Peevish means
A. Immovable
B. Guilty
C. Not guilty
D. Irritable

25. The band director was an expert a playing the piccolo.


Piccolo means
A. Small flute
B. Large flute
C. Small drum
D. Small triangle

26. The renter was remiss about the rent.


Remiss means
A. Timely
B. Negligent
C. Irritable
D. Impoverished
87

27. The old man was known for sapient knowledge.


Sapient means
A. Useless
B. Possessing wisdom
C. Perceptual
D. Limited

28. The inventor created several specious ideas to solve the problem.
Specious means
A. Inspired
B. Insufficient
C. Limited
D. Plausible

29. The tolerant attitude of the audience was appreciated.


Tolerant means
A. Tireless
B. Calm
C. Indulgent
D. Laborious

30. The verbose language used by the English teacher was tiresome to the class.
Verbose means
A. Wordy
B. Expressive
C. Limited
D. Punitive
88

Section 2 Reading Comprehension (Practice Test 3)

Reading 1

ROM (Range of Motion) of a patient may be needed to maintain a patient’s


mobility. Stretching of a patient’s muscles and joints may be more difficult that it
seems. When you stretch muscles and joints, you always have the chance of
injuring the patient. The medical principle of “Do No Harm” definitely applies with
ROM exercise.

There are multiple types of ROM. PROM, AAROM, and AROM are the key types
of ROM. PROM is range of motion that is passive. Basically, the patient offers
no assistance with PROM. Over-stretching with range of motion may lead to
injury to the joint and muscle. AAROM is active assistive range of motion, where
the patient helps some; however, the certified nursing assistant (CNA) offers the
additional assistance to complete the available range. AROM is active range of
motion that patients perform on their own. However, the patient may not have
full AROM of multiple joints. Contractions in debilitated patients are common in
the knees, ankles, shoulders, and pelvis.

Some stretches require AROM, followed by PROM to complete the available


range of motion. Never force a joint or muscle that exhibits abnormal resistance,
increased pain, or facial grimacing. Always observe the patient’s face while
performing range of motion exercises.

Never “bounce the tissue” when performing stretching, and hold your stretches
over sixty seconds or to the patient’s tolerance. Tissue fibers do not have the
opportunity to “stretch out” if the stretch is not held for sixty seconds. However, if
these fibers are over stretched a strain may be the result in some cases.

31. According to the reading what does PROM and AROM stand for?
A. Progressive Range of Motion and Active Range of Motion
B. Passive Range of Motion and Aggressive Range of Motion
C. Progressive Range of Motion and Aggressive Range of Motion
D. Passive Range of Motion and Active Range of Motion

32. What should certified nursing assistants never do while performing a stretch?
A. Hold stretch to patient’s tolerance
B. Bounce the tissue
C. Stop a stretch if a patient grimaces
D. Help patient perform the stretch

33. Which of the following was not a joint where contractures in debilitated
patients are commonly found?
A. Knees
B. Ankles
89

C. Wrist
D. Pelvis

34. When stretching a patient, you should be careful not to cause


A. an injury
B. fear
C. embarrassment
D. passive movement

35. Assuming a patient is unable to move a joint, which of the following ROM’s
would a certified nursing assistant perform?
A. AROM
B. AAROM
C. PROM
D. CPM

36. According to the article, how long does a stretch need to be held to be the
most effective?
A. 10 seconds
B. 30 seconds
C. 1 minute
D. 3 minutes

Reading 2

Interviews are often scary situations. It may feel as though your entire future
rests in how you behave and perform during those crucial 30 minutes to an hour.
That level of tension can create lingering doubts and worry. It is critical that you
not allow yourself to fall into that trap but remain above it and stay confident!

Think back over your past and try to remember all of your accomplishments. It
may help to review your resume and all of the highlights you’ve listed. Face it,
you are a talented individual and are more than capable of the job that you are
applying for. The interviewer just doesn’t know it yet, but don’t worry, you’ll show
him.

Visualize your success. Years ago, Sir Edmund Hillary was the first person to
climb to the top of Mt. Everest. He was later asked how it felt. He responded
that it felt just like it had every other time. When asked what he meant, he said
that he had already climbed Mt. Everest countless times in his mind. The actual
act of climbing the mountain was already a practiced event to him. He had
already visualized himself succeeding and that thought sustained him.

Be like Sir Edmund Hillary. Visualize yourself having the best interview in your
life and having the job offered to you. If you can visualize success, then you are
already halfway to your goal. You must remain confident in yourself, for if you
90

can’t be confident about yourself, who can? Your confidence begins with you.
Practice it for a while and you’ll see how contagious it can be.

You’ve heard the expression, “GIGO - Garbage in, Garbage out”. Only you can
control what you think about. Focus on positive, optimistic thoughts and you will
become positive and optimistic. Focus on thoughts of confidence, and you will
become confident. Each of us becomes what we think about. Make your choice
today about what you want to be and think on those thoughts!

37. The best title for this selection is


A. “Accentuate Your Positive Side”
B. “Being the Best You can Be”
C. “Succeeding at Interviews”
D. “Be Sir Hillary”

38. According to the article, which of the following is a key technique for
interviewing?
A. Dress appropriately
B. Visualize your success
C. Skim over the negative
D. Shake hands firmly with your interviewer

39. According to the article, what does “GIGO” mean?


A. If you focus on the positive, then you will present the perfect version of
yourself
B. If you think positive thoughts, then you will act positive during your
interview.
C. Your thoughts do not affect your interview
D. Get in, Get out with out making a mistake

40. According the article, which of the following would not be a step in preparing
for an interview?
A. Visualize yourself having a good interview
B. Review all of your accomplishments
C. Practice being confident
D. List out topics you want to avoid in your interview

41. The article stated that he had already visualized himself succeeding and that
thought sustained him. Sustained means
A. Alerted
B. Fed
C. Carried
D. Taught

42. The article supports which of the following?


A. Confidence comes from within.
B. If you fail at an interview, you did not try hard enough.
91

C. If you visualize climbing a mountain, you will be Sir Edmund Hillary.


D. You cannot practice to gain confidence.

Reading 3

Universal precautions are precautions taken to avoid contracting various


diseases and to prevent the spread of disease to those who have compromised
immunity. Some of these diseases include human immunodeficiency virus (HIV),
acquired immunodeficiency syndrome (AIDS), and hepatitis B (HBV).

Universal precautions are needed since many diseases do not display signs or
symptoms in their early stages. Universal precautions mean to treat all body
fluids/ substances as if they were contaminated. These body fluids include but
are not limited to the following blood, semen, vaginal secretions, breast milk,
amniotic fluid, feces, urine, peritoneal fluid, synovial fluid, cerebrospinal fluid,
secretions from the nasal and oral cavities, and lacrimal and sweat gland
excretions. This means that universal precautions should be used with all contact
with the public.

Special precautions must be taken when involved in treatment of patients. Avoid


touching lacerations or body fluids that are present, whenever possible. Wear
eye protection if there is a risk of fluid splashing into your eyes. Gown and gloves
will keep fluid from touching your clothes and skin. This may prevent you from
contracting a fatal disease. Be sure to wash your hands thoroughly after each
patient especially if you suspect body fluids have touched your skin.

If exchange of any body fluids does occur, seek medical attention immediately. If
a patient has HIV and body fluids are exchanged during a treatment, then viral
drugs may be given as soon as possible to reduce the risk of contracting HIV.
Protect yourself from injury and infection by seeking medical attention quickly.

43. What is the main topic of this article?


A. Contracting HIV
B. Preventative equipment
C. How to treat body fluids
D. The importance of universal precautions

44. Reading the article, which of the following professionals would most likely not
need to use universal precautions?
A. Nurse
B. Police Officer
C. Paramedic
D. Construction Worker

45. According to the article if you do get exposed to body fluid, what should you
do?
92

A. Try to find out if the person was infected with a deadly disease.
B. Seek medical attention immediately.
C. Report to your supervisor
D. Take care of the problem when you have time.

46. Which of the following situations increase your risk of contracting a fatal
disease?
A. Having sex with your monogamous spouse
B. Treating an abrasion while wearing latex gloves
C. Being in an altercation with a suspect where a laceration occurs
D. Giving a HIV infected person a hug.

47. Universal precautions mean that you


A. Touch no one.
B. Treat everyone’s body fluid as if it were contaminated with a fatal disease.
C. Wear gloves to suture a laceration only if you know the person has HIV.
D. Treat everyone as if they do not have a disease until proven other wise

48. Which of the following statements are false?


A. If you are exposed to HIV, there is nothing you can do.
B. Fatal diseases can be transmitted through blood only.
C. All disease display symptoms in early stages.
D. If you wear eye protection, you may avoid being contaminated with a
fatal disease.

Reading 4

Ask yourself the question, “Who is the most important person to my college
career in my school?” The answer should be your guidance counselor. Why?
Because the guidance counselor has access to information that could save you
thousands of dollars in college tuition and fees. In addition, they have the
resources for career preparation and scholarships. Millions of students go
through four years of high school without ever meeting their guidance counselor.
Consequently, a guidance counselor responding to a college admission officer’s
phone call who does not know the student may not adequately represent to the
college acceptance board that student’s accomplishments.

It is critical that the prospective college applicant should meet their guidance
counselor. If possible, a college bound student should meet their guidance
counselor during their freshman year and begin to discuss interest in various
fields and colleges. If expectations are unrealistic, the guidance counselor will
identify discrepancies in your academic performance that may not allow top
college admission success. This may give you the encouragement needed to put
more effort into your academics. Moreover, the guidance counselor will now put a
face with a name during college preparation and scholarship searches.
93

A high school guidance counselor should have resources that identify key
scholarships and scholarships that are not obvious during your scholarship
search that could add up. Ask for a list of these types of scholarships when
entering your junior year of high school. Some scholarships require preparation
in the junior year of high school. Corporate scholarship applicants often must
apply and then be tracked during their senior year to determine if they are the
appropriate candidates.

Guidance counselors can offer excellent practice interviews to prepare for a


scholarship or college admission interview. They should identify statements or
mannerisms that do not flow during the interview. They may identify information
that could be reworded during the real scholarship or college admission
interview. In addition, they can review your high school track and determine if it
is going to help you with the college of your choice. For example, certain
courses are required for graduation according to state law. Students that transfer
during high school to other high schools could run the risk of missing a key
course. Graduation may be delayed, if a state required course is by-passed
accidentally. Also, they may suggest electives like computer science that may
assist you for the rest of your life.

At a minimum, high school seniors should meet with the guidance counselor at
least once for a discussion about possible career interests and college funding.
Remember the old expression that the “early bird gets the worm.” Some
scholarships are on first come, first serve basis. Ask yourself the question? Will
I look more professional if my scholarship application arrives at the latest
possible date or the earliest? Reviewers may focus on the scholarship
application that shows forethought and planning for the earliest presentation.

Your high school guidance counselor may not be the best educator in your
school; however, the counselor is your most valuable tool when applying to
college. Attempt to obtain every available ounce of knowledge and college
preparation material from your high school guidance counselor. The information
shared with you by your high school guidance counselor may determine a career
choice, open new possibilities, and allow realistic expectations for college
admission.

49. What is the main idea of this article?


A. A guidance counselor is a great resource for college.
B. College scholarships and deadlines
C. Finding information on college
D. Meeting your guidance counselor

50. According to the article, what is the minimum number of times high school
seniors should meet with their guidance counselors?
A. weekly
B. 4 times per year
94

C. at least once
D. daily

51. According to the article, when is the best time to send in an application?
A. Towards the end of the deadline to show you put thought into your
application.
B. In the middle of the acceptance phase to not seem over eager
C. At the beginning of the acceptance phase to show forethought and
planning while you prepared to apply for the scholarship
D. When you get to it

52. Which of the following services do most guidance counselors offer their
students?
A. practice for interviews
B. information about scholarships
C. discussion about possible career choices
D. all of the above

53. The article states that they should identify statements or mannerisms that do
not flow during the interview. Mannerism means
A. Idiom
B. Characteristic
C. Difficulty
D. Exchange

54. Guidance counselors are referred to by which of the following terms in the
article?
A. A valuable tool
B. College admissions officer
C. Corporate scholarship decider
D. A priceless jewel

Reading 5

Over the next 10-20 years the number of baby boomers requiring medical
intervention is going to reach over 7 million people. The need for over 50,000
new radiologic technologists will be created in the next 20 years, because of the
baby boomers requiring medical intervention. Job security will not be the concern
of radiologic technologists in the future. Radiologic technologists who are trained
in multiple fields will be offered the best opportunities. In 2000, there were about
167,000 radiologic technologists employed in the United States according the
Bureau of Labor Statistics.

Pick up a newspaper and you will probably find 3-5 offers regarding radiological
technologist employment. The opportunities for travel and job security can be
found in a radiological technologist position. In addition, experienced radiological
95

technologists are often asked to serve on staff committees at many medical


facilities. The opportunity for patient care and medical experience are found in a
radiological technologist position.

Radiological technologists can specialize in computed tomography (CT) and


magnetic resonance imaging (MRI). Additional specialties include diagnostic
sonographers, nuclear medicine technologists, and cardiovascular technologists.
Moreover, even some MD’s have started at the radiological technologist position
to learn the inter-workings of a medical facility. The radiological technologist may
even use their degree as a springboard for a physician assistant position, which
would require going to (PA) school. The chances for advancement and job
satisfaction are available for those individuals who offer compassionate and
medically responsible care. However, there are many challenges in the job of a
radiological technologist. These challenges include time management, job
hazards, overbearing senior staff, and difficult patients.

55. According to the article, which of the following is a benefit to becoming a


radiological technologist?
A. Time management
B. Job hazards
C. Job security
D. Overbearing senior staff

56. Which of the following is not a specialized field of radiology?


A. Computed tomography
B. Magnetic resonance imaging
C. Diagnostic sonographers
D. Physicians Assistant

57. What reason does the article give for increased need for radiological
technologist for the next twenty years?
A. Improved technology
B. Decreased number of radiology schools
C. Baby boomers requiring medical intervention
D. Retiring of older radiological technologist

58. According to the article about how many radiological technologists were
employed in 2000?
A. 165,000
B. 167,000
C. 176,000
E. 1,760,000

59. The article states that the radiological technologist may even use their degree
as a springboard… Springboard means
A. Jump start
96

B. Diving board
C. Resting place
D. Career guide

60. How many classified ads per day offering radiological technologist jobs are in
the average newspaper according to the article?
A. 1-2
B. 3-5
C. 6-9
D. 12-15

Answer Key for Word Knowledge Section 2 (Practice Test 3)

1. A
2. D
3. B
4. C
5. B
6. B
7. A
8. C
9. D
10. D
11. C
12. D
13. A
14. B
15. A
16. C
17. A
18. A
19. D
20. B
21. A
22. C
23. B
24. D
25. A
26. B
27. B
28. D
29. C
30. A

Answer Key for Reading Comprehension Section 2 (Practice Test 3)

31. D
97

32. B
33. C
34. A
35. C
36. C
37. C
38. B
39. B
40. D
41. C
42. A
43. D
44. D
45. B
46. C
47. B
48. C
49. A
50. C
51. C
52. C
53. B
54. A
55. C
56. D
57. C
58. B
59. A
60. B
98

Section 3 Science (Practice Test 3)

Answer each problem carefully. The correct answers are found after the
section. You are allowed 60 minutes to complete this test.

1. When the chromosomes line up in mitosis this is known as which phase?


A. Telophase
B. Anaphase
C. Metaphase
D. Prophase

2. Which cellular organelle contains enzymes that are considered digestive?


A. Golgi Apparatus
B. Lysosomes
C. Nucleus
D. Ribosomes

3. Organs repair themselves through a process of?


A. Meiosis
B. Mitosis
C. Cellular differentiation
D. Transformation

4. Which of the following is considered a model for enzyme action?


A. Lock and Key model
B. Enzyme interaction model
C. Transformation model
D. Transcription model

5. Which of the following statements about enzymes is not true?


A. Enzymes are catalysts.
B. Almost all enzymes are proteins.
C. Enzymes operate most efficiently at optimum pH.
D. Enzymes are destroyed during chemical reactions.

6. Which of the following statements about prostaglandins is not true?


A. Prostaglandins promote inflammation.
B. Prostaglandins can only constrict blood vessels.
C. Prostaglandins are made in the renal medulla
D. Prostaglandins can lead to pain and fever.

7. Cholesterol that is known as (LDL) stands for:


A. Low-density lipoproteins
B. Low-density lysosomes
C. Level-density lipoproteins
D. Level-density lysosomes
99

8. Hardening of the arteries is known as:


A. Atherosclerosis
B. Venous narrowing
C. Micro-circulation
D. Hypertension

9. The top number on a blood pressure reading indicates:


A. Diastolic pressure
B. Transient pressure
C. Optimum pressure
D. Systolic pressure

10. Litmus paper that is blue will turn/stay _____ in the presence of a strong
base.
A. Orange
B. Red
C. Blue
D. Green

11. Breathing properly requires the presence of what compound that affects
surface tension of alveoli in the lungs?
A. Potassium
B. Plasma
C. Surfactant
D. Sodium Chloride

12. Which of the following is not considered a function of the kidneys?


A. Secretion
B. Reabsorption
C. Transport
D. Filtration

13. The functional unit of the kidney is known as?


A. Medulla
B. Glomerulus
C. Pyramid
D. Nephron

14. Which of the following formulas indicate the ideal gas law?
A. PV= knT or PV=RnT
B. V=kT
C. PV = k
D. KTV =PR

15. Which of the following is not considered a characteristic or property of a gas?


A. Volume
100

B. Mass
C. Pressure
D. Particles

16. Which of the following is described in the definition: An object immersed in a


fluid is buoyed up by a force equal to the weight of the fluid displaced by the
object?
A. Archimedes’ principle
B. Charles’ law
C. Boyle’s law
D. Anderson’s principle

17. Liquids that evaporate quickly are known as _____ liquids.


A. Viscous
B. Volatile
C. Evaporative
D. Transient

18. High frequency sound waves are known as:


A. Fundamental waves
B. Overtones
C. Consonance waves
D. Dissonance waves

19. The first American to win a Nobel Prize was ______ for measuring the speed
of light.
A. Albert Einstein
B. Albert Michelson
C. Grimaldi
D. Thomas Young

20. What anatomical structure connects the stomach and the mouth?
A. Trachea
B. Spinal column
C. Hepatic duct
D. Esophagus

21. The movement of food through the intestines is known as:


A. Peristalsis
B. Ileum translation
C. Microvilli propulsion
D. Flexure propulsion

22. The enzyme maltase does the following:


A. Breaks down lactose to glucose
B. Turns glucose into maltose
101

C. Breaks down maltose to glucose


D. Turns glucose into lactose

23. High levels of bilirubin in the blood stream can result in:
A. Uric acid overexposure
B. Jaundice
C. Bile salt production
D. Hepatic mutation

24. The symbol B on the periodic table stands for:


A. Beryllium
B. Boron
C. Barium
D. Berkelium

25. The symbol Mn on the periodic table stands for:


A. Magnesium
B. Molybdenum
C. Manganese
D. Margon

26. The symbol Ca on the periodic table stands for:


A. Calcium
B. Carbon
C. Cobalt
D. Chlorine

27. The symbol Br on the periodic table stands for:


A. Beryllium
B. Boron
C. Barium
D. Bromine

28. Vinegar is also known as:


A. Acetic acid
B. Acetone acid
C. Sulfuric acid
D. Ascorbic acid

29. A Lewis acid is a/an ____ pair acceptor.


A. Neutron
B. Proton
C. Electron
D. Ion

30. _____ reactions produce heat.


102

A. Endothermic
B. Exothermic
C. Hydrogen
D. Buffered

31. Which of the following match the definition: The pressure (P) is inversely
proportional to the volume (V) of a gas.
A. Archimedes’ principle
B. Charles’ law
C. Boyle’s law
D. Anderson’s principle

32. The combination of sodium and chlorine to form NaCl is considered a/an ___
bond.
A. Ionic
B. Covalent
C. Hydrogen
D. Metallic

33. Outer shell electrons are known as ______ electrons.


A. Hybrid
B. Valence
C. Vector
D. Transitional

34. Forces can be indicated on graph paper by the use of _____.


A. Empirical rules
B. Interaction coefficients
C. Variables
D. Vectors

35. P1V1 = P2V2 represents:


A. Archimedes’ principle
B. Charles’ law
C. Boyle’s law
D. Anderson’s principle

36. Which of the following is not considered a primary color of light?


A. Green
B. Blue
C. Red
D. Yellow

37. The unit of charge is called the _____.


A. Newton
B. Coulomb
103

C. Joule
D. Watt

38. A/An ______ is a device specifically designed to measure current.


A. Ammeter
B. Cyclotron
C. Resistor
D. Capacitor

39. The two bones found in the area between the knee and ankle in humans are
known as:
A. Femur and Tibia
B. Fibula and Tibia
C. Ulna and Tibia
D. Radius and Tibia

40. _____ is the current flow of one coulomb per second.


A. Ampere
B. Joule
C. Watt
D. Kilowatt

41. Which of the following is considered a component of lipids?


A. Plasma cells
B. Fatty acids
C. Nucleic acids
D. Zinc

42. Down’s syndrome affects chromosome ____.


A. 13
B. 15
C. 21
D. 23

43. Blood enters the lungs from which chamber of the heart?
A. Right atrium
B. Left atrium
C. Right ventricle
D. Left ventricle

44. Excessive consumption of alcohol is most likely to damage which organ of the
body over a long period of time?
A. Kidney
B. Liver
C. Pancreas
D. Gallbladder
104

45. Which of the following is not considered a type of radiation ray?


A. Gamma
B. Beta
C. Alpha
D. Infrared

46. A molecule of hemoglobin can hold how many molecules of oxygen in the
blood for transport?
A. 2
B. 4
C. 6
D. 8

47. Which of the following best describes the biomechanics of breathing?


A. Pump handle motion
B. Lever action
C. Inspiration
D. Expiration

48. Animals that eat meat almost exclusively are known as:
A. Herbivores
B. Carnivores
C. Arthropods
D. Prolific organisms

49. The physical expressions of a gene are known as an organism’s:


A. Transcription
B. Genotype
C. Phenotype
D. Translation

50. A ball is traveling at a constant velocity of 50 m/s and has been traveling for
over 2 minutes. What is the ball’s acceleration?
A. 0
B. 25 m/s
C. 25 m/s2
D. 50 m/s2

51. Neurons connect together at a ______.


A. Synergy
B. Terminal site
C. Docking station
D. Synapse

52. Which of the following is another word for the kneecap in the human body?
105

A. Pisiform
B. Meniscus
C. Popliteal bursa
D. Patella

53. Which of the following describes the shoulder joint?


A. Ball and socket joint
B. Saddle joint
C. Hinge joint
D. Pivot joint

54. The organ of Corti is found in what area of the body?


A. Mouth
B. Ear
C. Nose
D. Lungs

55. The condition of rickets is associated with a deficiency in which vitamin?


A. A
B. C
C. D
D. Z

56. A steroid is considered a ______.


A. Lipid
B. Protein
C. Enzyme
D. Weak acid

57. The X cranial nerve is the ____ nerve.


A. Abducens
B. Hypoglossal
C. Facial
D. Vagus

58. Which of the following pH ranges is a strong base?


A. 1.3-2.0
B. 4.5-5.2
C. 7.1-9.0
D. 11.2-12.0

59. Which chamber of the heart pumps blood to the systemic circulation?
A. Left Atrium
B. Right Atrium
C. Left Ventricle
D. Right Ventricle
106

60. Which of the following formulas indicates Newton’s second law of motion?
A. F = ma
B. F = mva
C. v = d/t
D. p = mv
107

Answer Key for Science Section 3 ( Practice Test 3)

1. C
2. B
3. B
4. A
5. D
6. B
7. A
8. A
9. D
10. C
11. C
12. C
13. D
14. A
15. D
16. A
17. B
18. B
19. B
20. D
21. A
22. C
23. B
24. B
25. C
26. A
27. D
28. A
29. C
30. B
31. C
32. A
33. B
34. D
35. C
36. D
37. B
38. A
39. B
40. A
41. B
42. C
43. C
44. B
108

45. D
46. B
47. A
48. B
49. A
50. A
51. D
52. D
53. A
54. B
55. C
56. A
57. D
58. D
59. C
60. A

Vous aimerez peut-être aussi